Jump to content

Wikipedia:Reference desk/Science: Difference between revisions

From Wikipedia, the free encyclopedia
Content deleted Content added
Line 823: Line 823:
:I've never heard of this particular author - so I'm not going to vouch for the entirety of everything else he's written (in particular, his unique terminology "exergy" is not commonly used in other books on statistical physics) - but I can recommend several other great books on statistical mechanics if you're interested.
:I've never heard of this particular author - so I'm not going to vouch for the entirety of everything else he's written (in particular, his unique terminology "exergy" is not commonly used in other books on statistical physics) - but I can recommend several other great books on statistical mechanics if you're interested.
:[[User:Nimur|Nimur]] ([[User talk:Nimur|talk]]) 20:10, 24 March 2016 (UTC)
:[[User:Nimur|Nimur]] ([[User talk:Nimur|talk]]) 20:10, 24 March 2016 (UTC)
::But with this formulation he calculates entropy wrong way. He must take <math>j=1,2,...,m</math>. If particle states are 0 and 1 <math>j=1,2</math>.
::But with this formulation he calculates entropy wrong way. He must take <math>j=1,2,...,m</math>. If particle states are 0 and 1 , <math>j=1,2</math>.

Revision as of 20:23, 24 March 2016


Welcome to the science section
of the Wikipedia reference desk.
Select a section:
Want a faster answer?

Main page: Help searching Wikipedia

   

How can I get my question answered?

  • Select the section of the desk that best fits the general topic of your question (see the navigation column to the right).
  • Post your question to only one section, providing a short header that gives the topic of your question.
  • Type '~~~~' (that is, four tilde characters) at the end – this signs and dates your contribution so we know who wrote what and when.
  • Don't post personal contact information – it will be removed. Any answers will be provided here.
  • Please be as specific as possible, and include all relevant context – the usefulness of answers may depend on the context.
  • Note:
    • We don't answer (and may remove) questions that require medical diagnosis or legal advice.
    • We don't answer requests for opinions, predictions or debate.
    • We don't do your homework for you, though we'll help you past the stuck point.
    • We don't conduct original research or provide a free source of ideas, but we'll help you find information you need.



How do I answer a question?

Main page: Wikipedia:Reference desk/Guidelines

  • The best answers address the question directly, and back up facts with wikilinks and links to sources. Do not edit others' comments and do not give any medical or legal advice.
See also:


March 19

Name for type of spring

What's the name for this[1] kind of "pyramid-shaped" spring?

In case there's no special name for it, what's best keywords that I can use find it on eBay or Amazon? I tried "pyramid shaped spring" but it doesn't do the job. Johnson&Johnson&Son (talk) 06:35, 19 March 2016 (UTC)[reply]

"Conical spring" or "tapered spring" find lots of them on google. DMacks (talk) 06:54, 19 March 2016 (UTC)[reply]
A similar spring is sold in Sweden & Norway as a "Tankspiral" whose description translates as Effectively stops the petrol thief. Prevents theft by hose of fuel via the filler pipe. Easily mounts in the fuel filler pipe. AllBestFaith (talk) 13:15, 19 March 2016 (UTC)[reply]
It's a "conical" or "tapered" spring. If there was such a thing as a "pyramid-shaped spring" it would have a bunch of square corners. The geometric shape that's a circular shape that gradually tapers to a point is called a "Cone" - hence "conical spring". A spring that actually did have square corners wouldn't work very well because all of the twisting force would end up concentrated in the corners. SteveBaker (talk) 16:53, 19 March 2016 (UTC)[reply]
The spring in your Picture does not have a standard, common form nor does it look like a common spare part so no Wonder you can not find it online anywhere. Manufacturers who use such special formed springs will order them as mass product from specialized companies that massproduce such springs in almost any form you order. Then the ordering Manufacturer usually gives it a functional name in context of its use. --Kharon (talk) 16:53, 19 March 2016 (UTC)[reply]
Did you even read AllBestFaith's post (above) which seems to be the best description yet? Akld guy (talk) 04:51, 20 March 2016 (UTC)[reply]
The original poster may have a larger size in mind, but conical springs are very common in battery compartments for the common sizes AAA, AA, C, and D. They are typically used as the negative contact and also press the battery against the positive contact. See this one and this one and this one, and this one, for example.
Curiously, the article spring (device) includes a red link to "conical spring" but outside of that passage it uses the phrase only in a single sentence, which is garbled. However, coil spring includes illustrations of conical coil springs. --69.159.61.172 (talk) 07:25, 20 March 2016 (UTC)[reply]
@Akld guy Again, there is no standard on conical springs. Not form nor size nor material. Any such spring is proprietary. So bite me :p. --Kharon (talk) 22:30, 21 March 2016 (UTC)[reply]

Did a cheapness always allows to satisfy much needs of more people?

Did a cheapness always allows to satisfy much needs of more people?--83.237.192.157 (talk) 09:36, 19 March 2016 (UTC)[reply]

So, did a cheapness of electricity and natural gases allows to satisfy much needs of more people?--83.237.192.157 (talk) 09:45, 19 March 2016 (UTC)[reply]

It sounds like you're talking about Supply and demand. Ian.thomson (talk) 10:04, 19 March 2016 (UTC)[reply]
I don't know. Did a combinations of electricity and natural gases always allows to save an electricity and natural gases?--83.237.192.157 (talk) 10:24, 19 March 2016 (UTC)[reply]
So, did technical methods - technical levels of application and use are makes a cheapness?--85.141.236.80 (talk) 11:10, 19 March 2016 (UTC)[reply]
So that, did technical methods - technical levels of application and use are makes a economy?--85.141.236.80 (talk) 11:13, 19 March 2016 (UTC)[reply]
Please try and write clearer English. There's no such thing as 'a cheapness'. And there's a difference between 'an economy' - the whole financial situation of a country or other entity - and 'making economies' - making savings. AlexTiefling (talk) 11:23, 19 March 2016 (UTC)[reply]
Thanks. It seems me, that the levels of complexity of technique are always decides all.--85.141.236.80 (talk) 11:28, 19 March 2016 (UTC)[reply]
It is necessary to identify cases in which the levels of complexity of technique are not decides all.--85.141.236.80 (talk) 11:37, 19 March 2016 (UTC)[reply]
Did the most simplest technical approach to application and use of natural gases is justified in my country USSR-Russia?--85.141.232.111 (talk) 12:20, 19 March 2016 (UTC)[reply]
We can not understand you. Please try [ https://ru.wikipedia.org/ ]. --Guy Macon (talk) 12:28, 19 March 2016 (UTC)[reply]
I'm try again. If technical methods - technical levels are allows to economy, did the most simplest technical approach of application and use of natural gases makes a cheapness?--85.141.232.111 (talk) 13:17, 19 March 2016 (UTC)[reply]
Did technogenesis of economy of the USSR-Russia is insufficient?--83.237.222.31 (talk) 15:16, 19 March 2016 (UTC)[reply]

PLEASE NOTE: We are not understanding your questions because either your English isn't good enough - or you're using a bad translation service. You need to ask your question on the Russian Wikipedia Help/Reference desk. We can't help you here. SteveBaker (talk) 16:49, 19 March 2016 (UTC)[reply]

If the question were coming from France or Esperantujo, I'd invite them to ask in their first language and I'd translate. —Tamfang (talk) 23:57, 20 March 2016 (UTC)[reply]
Thanks for your help. Of course, a technogenesis of economy of the USSR-Russia did small, I think it did so, because economy of the USSR-Russia did using a natural gases and did saving an electricity and natural gases, that it is what about I’m did interesting here.--Alex Sazonov (talk) 18:28, 19 March 2016 (UTC)[reply]
When a commodity becomes cheaper, people can afford proportionally more of it. However, people who compete to produce that commodity lose all income once the price decreases below their cost of production. For example, as described by the Zapatistas 20 years ago, and recently discovered by some Trump-leaning media in the U.S., the availability of federally subsidized corn from the U.S. via NAFTA caused indigenous peasants in Mexico to lose their ability to support themselves on their land, driving them to the cities and in turn driving the workers they displaced into the U.S. as illegals, where they displaced yet others... there are many solutions suggested for problems like this, provided that they involve neither the repeal of corn subsidies nor the repeal of NAFTA. Wnt (talk) 21:33, 19 March 2016 (UTC)[reply]
Did it makes sense to economies its development, that is did it makes sense to economies a development of its technogenesis? My country USSR-Russia did making economies its development, because its economy did using a natural gases and did saving an electricity and natural gases.--83.237.207.51 (talk) 11:27, 20 March 2016 (UTC)[reply]
It more always depends on to economies on or to economies its.--83.237.207.51 (talk) 13:14, 20 March 2016 (UTC)[reply]
So that, what technogenesis leads to a cheapness, it’s a small or high technogenesis?--83.237.207.51 (talk) 14:30, 20 March 2016 (UTC)[reply]
New technology often makes a commodity cheaper; for example, John D. Rockefeller made himself rich by making oil cheaper by new techniques of extraction and refining. On the other hand, a change in the relative prices of two or more commodities can make a technology (new or old) more practical. —Tamfang (talk) 23:59, 20 March 2016 (UTC)[reply]
Thanks a lot! Did the banking on natural gases justified in my country USSR-Russia, that is did the most simplest technical approach of application and use of natural gases justified in my country USSR-Russia, if a cheapness did not come to my country USSR-Russia? Maybe a cost of natural gas always decides all, so that maybe a cost of natural gas is too high?--85.141.237.148 (talk) 08:14, 21 March 2016 (UTC)[reply]

Sword-billed hummingbird preening

The Sword-billed hummingbird has a bill longer than its body. How does it preen itself? (Please feel free to add the answer to the article). Andy Mabbett (Pigsonthewing); Talk to Andy; Andy's edits 12:08, 19 March 2016 (UTC)[reply]

[ http://astronomy-to-zoology.tumblr.com/post/43434261341/a-sword-billed-hummingbird-ensifera-ensifera ] has a video of a sword-billed hummingbird preening with his feet. --Guy Macon (talk) 12:31, 19 March 2016 (UTC)[reply]
You just beat me, Guy. Here's another: [2]. Excellent question, Andy! Snow let's rap 12:37, 19 March 2016 (UTC)[reply]
Also *cough* that detail is already found in the article. ;) Still glad you asked though. Every day you learn a new ornithological fact is a day you are a little closer to nirvana. :D Snow let's rap 12:43, 19 March 2016 (UTC)[reply]

Thanks you, all. I missed that in the article, because I searched for the string "preen". I've fixed it now. Andy Mabbett (Pigsonthewing); Talk to Andy; Andy's edits 16:06, 19 March 2016 (UTC)[reply]

Hi all. Great question. Loved the videos. Do we have a policy/guideline on inserting a link to videos (I am thinking only of animal behaviour here) within the text, rather than "External links" section? This may not be the place to raise this question - it it is not, where might this place be? DrChrissy (talk) 15:15, 21 March 2016 (UTC)[reply]
I just read WP:External links which explains this. Please disregard the question. DrChrissy (talk) 15:38, 21 March 2016 (UTC)[reply]

I'm looking for the simplest Serial Wire Debug port pinout possible. The best one I've found (best in terms of minimal pins) so far is this 10 pin version[3].

1. How come all these SWD pinout are so needlessly complex? Doesn't SWD just have two data pins? Ground, Vcc, and the two data pins would be only 4 pins in total. All the pinouts I've found are at least double that.

2. Is there a common-ish SWD header that's less than 10 pins?

3. I noticed that the standard 10 pin SWD header has a reset pin. Is this actually necessary? Doesn't the SWD protocol already implement a reset command? Or does this reset pin serve as a backup hard reset as opposed to the SWD's soft reset? Johnson&Johnson&Son (talk) 15:37, 19 March 2016 (UTC)[reply]

These are the only pins required:
  • Clock
  • Data (2-way)
  • Ground

In the question above, you assumed that power was required, but it is not.

These are optional, but often useful:
  • Power to target
  • A second ground pin to reduce noise
  • Reset target
  • Serial Wire Output - not required but allows extra trace capabilities
  • Target reference voltage - voltage from target that sets the output logic levels sent to the target (avoids feeding 0-5V signals to a board that is expecting 0-3.3V...)
  • Key - unused pin to prevent connector being plugged in backwards.
  • Various extra signals so that the same connector supports SWD and JTAG.
A reset pin will hard reset a hung/crashed system. A reset command assumes that the system is working well enough to receive the command. --Guy Macon (talk) 16:40, 19 March 2016 (UTC)[reply]
If you are designing a board, I recommend using one of the connectors listed here: [ http://www.keil.com/support/man/docs/jlink/jlink_connectors.htm ]. While there are no real standards, those three are fairly widely used. --Guy Macon (talk) 16:59, 19 March 2016 (UTC)[reply]
The article you linked to covers all of this. It says "There are no official standards for JTAG adapter physical connectors." - so your idea that this is a "standard 10 pin header" isn't correct. It also says: "Those connectors tend to include more than just the four standardized signals (TMS, TCK, TDI, TDO). Usually reset signals are provided, one or both of TRST (TAP reset) and SRST (system reset). The connector usually provides the board-under-test's logic supply voltage so that the JTAG adapters use the appropriate logic levels. The board voltage may also serve as a "board present" debugger input. Other event input or output signals may be provided, or general purpose I/O (GPIO) lines, to support more complex debugging architectures."
The actual "serial debug" part of the JTAG interface is indeed just two wires - TxD,RxD (with an "assumed" common ground connection) - but when you're debugging a circuit with a JTAG instrument, you generally want to do other things - supplying power, ground and a system clock and doing a hard-reset on the system are obvious requirements.
The article also lists the common connector types with the numbers of pins - so the article also answers (2).
So if you're using a standard JTAG tester - you'll need to follow whatever pinout it demands. If you're inventing your own JTAG debugger (or using the serial standards of JTAG for something else) - then two wires plus ground will suffice - and then you're at liberty to use any 2 (or preferably 3) pin connector you want. SteveBaker (talk) 16:44, 19 March 2016 (UTC)[reply]

Do you need to burn fuel to decelerate a spaceship when landing?

Couldn't you just reach a planet parallel and in the opposite direction of its orbit, moving in a parabolic arc, and then land softly? You'll need a precise calculation, but that would reduce the amount of fuel you need for your rocket.--Scicurious (talk) 18:02, 19 March 2016 (UTC)[reply]

The laws of physics run backwards just the same. If what you said could happen then things on the earth could just lift off and go into space without fuel. Dmcq (talk) 18:30, 19 March 2016 (UTC)[reply]
I am not sure that's the same case backwards. In my scenario the spaceship approaches a planet and has lots of kinetic energy and the planet brakes it due to its gravity. In the inverse scenario, energy could not just appear to accelerate an object and escape the surface of a planet. --Scicurious (talk) 18:36, 19 March 2016 (UTC)[reply]
You can land on a small asteroid with zero extra fuel but you cannot just reach a planet already stopped. The planet has a gravity well and any inert object going down gains many kilometers per second speed whether it wants to or not. It is possible to land with zero fuel with a skip reentry and then parachutes (and airbags if needed). Sagittarian Milky Way (talk) 19:16, 19 March 2016 (UTC)[reply]
The "laws of physics are reversible" argument should be usable to reverse the ever-popular "slingshot manouver" - used to speed up a spacecraft by stealing some kinetic energy from a suitable planet. I have no idea how - but it seems like this should be able to slow you down...somehow. SteveBaker (talk) 01:25, 20 March 2016 (UTC)[reply]
I believe a slingshot works because the ship approaches the planet slowly, and thus has it's gravity act on it for a long time, to accelerate it, then leaves the planet quickly, thus having less time for the planet's gravity to act on it to decelerate it. To reverse that, I'd expect you would need to reverse the force of gravity. However, skimming the atmosphere might work to slow it down so it leaves to planet more slowly, so you could perhaps get a reverse slingshot that way, if you could slow it down more than it accelerated as it approached the planet. StuRat (talk) 01:45, 20 March 2016 (UTC)[reply]
If you imagine the planet and spaceship as hard, elastic objects like billiard balls, and if there is no atmospheric braking involved/ then a slingshot maneuver produces exactly the same results as if the spaceship simply bounced off the planet. This is a necessary result of the conservation of energy and momentum. The reason that these maneuvers are sometimes useful is that we can choose where in the planet's orbit the spacecraft will approach it, and therefore which way it will "knock" the spacecraft. The only reasons to go around the planet instead of bouncing off it are the important facts that planets and spaceships are not hard, elastic objects and that there may be an atmosphere that you usually want to avoid. --69.159.61.172 (talk) 07:39, 20 March 2016 (UTC)[reply]
Not necessarily from one planet to the next. It's a question of rotational speed and timing the tangent, lagrangian point and rotation of the other body. You would need to slow down enough from the leaving body (or passing body so the velocity imparted by the arriving body matched its rotational speed. It's easier calculate if you don't worry about the launch bit. How far away from a planet without an atmosphere does spaceship have to be to reach the same rotational speed as the body. It's tricky because as was pointed out, it accelerates into the body and the precise point it reaches rotational velocity it has to be at the surface or crash or miss. --DHeyward (talk) 02:54, 24 March 2016 (UTC)[reply]
This is implied by what 69.159.61.172 wrote, but to make it clear, you can use a slingshot maneuver to slow down, but not relative to the object you're slingshotting around, only relative to some other object. (And even if you use a slingshot around some other planet to slow to near rest relative to the planet you want to land on, you will still hit the surface at roughly the escape velocity if there's no atmospheric friction and you don't fire your rockets.) -- BenRG (talk) 09:12, 20 March 2016 (UTC)[reply]
Aha! "not relative to the object you're slingshotting around" - thanks! Very perceptive answer. SteveBaker (talk) 15:32, 20 March 2016 (UTC)[reply]
With a thick atmosphere or oceans, those can be used for deceleration, but the tricky part is not destroying the ship in the process. StuRat (talk) 20:05, 19 March 2016 (UTC)[reply]
The problem with a parabola is that the arc is level at the point of release and not level at the point of impact. Things are stationary when you let go of them, not when they hit the ground; and if shot up from the ground, they become stationary at some point in mid-air (assuming they don't get to orbit). To put it another way, the speed decreases when it is a speed upward against a downward acceleration, but it increases when it is a speed downward added to more downward acceleration. So there is no way to plot the orbit to land softly on the planet, though of course a clever glider (aka "flying brick", as the Space Shuttle was lovingly called) could make use of the air to slow down and control a landing. Wnt (talk) 21:12, 19 March 2016 (UTC)[reply]
As our gravity assist article explains, you can indeed use it to slow down -- just go round the planet the other way. Aurora (novel) has a fictional example (which also includes some hardcore aerobraking. HenryFlower 07:24, 20 March 2016 (UTC)[reply]

The short answer to the original question is that you can't do this because the planet's gravity always pulls you toward the planet. As soon as you reach a position where you are moving toward the planet, it will therefore accelerate you toward it. In fact, if the planet has no atmosphere and you reach it without rocket or similar propulsion, you will always crash-land at the planet's escape velocity or faster. --69.159.61.172 (talk) 07:32, 20 March 2016 (UTC)[reply]

Actually you can. There are many points such as the lagrangian, geostationary, slingshot, etc, that all use gravity to maintain position. The trick is the large body like earth has air and friction. Without that, you could conceivably time your entry velocity so that you met the rotational velocity at the surface. That's about 1,000 mph at the surface and space vehicles slow down to that velocity so the land where they want. You would need another body to slow down the craft to hit proper entry so that your velocity into the well matched the rotation. This was huge in free return paths from the moon and missing that entry azimuth and velocity meant death. Apollo ships did not need propellant to return to earth and their paths were designed so they could suffer primary and secondary propulsion and still enter the atmosphere at the correct angle and speed. Now, an atmosphere that travels at roughly the same rotation as the body makes it a heat problem to solve. Take atmosphere away but still have a rotating body, it's a needle in a haystack. There is a solution but it's not very error friendly. People don't survive crashes at high speed and matching rotation could be nasty when the surface is moving 1000 mph relative to the fixed stars. Here's a NASA paper regarding their launch windows for Apollo and choices they made [4]. In missions where they inserted into lunar orbit the mechanics were a bit different than free return figure 8 (a la apollo 13). --DHeyward (talk) 02:29, 24 March 2016 (UTC)[reply]

To be a little clearer, landing on the moon with no atmosphere would be very difficult because it's tide locked with earth and rotates once in 28 days (very slow). You would have to using something like jupiter or the sun to slow you down, hope there was tangent where the sun/planets didn't take over (or the earth). That solution may not exist depending on mass, lagrangian points and other things. And no there is no equation, it's numerical. --DHeyward (talk) 02:41, 24 March 2016 (UTC)[reply]

Ritalin, similar chemically similar to cocaine?

Is Ritalin very close to cocaine? Does it matter if you are trying to estimate the effect of it? After all, two substances can be chemically very close, but have, biologically, a completely different effect on an organism? Ignoring social taboos relating to cocaine use, would it be feasible to substitute therapeutic Ritalin use for cocaine? --Llaanngg (talk) 18:21, 19 March 2016 (UTC)[reply]

Some similar chemicals act biologically identical, while others behave quite differently (for example, if the difference is in the "key" which allows it to enter your cells). StuRat (talk) 20:02, 19 March 2016 (UTC)[reply]
Neither Ritalin nor cocaine need to enter cells to have their effects. Fgf10 (talk) 23:07, 19 March 2016 (UTC)[reply]
My response was to the 3rd Q, which doesn't ask about specific drugs. StuRat (talk) 23:28, 19 March 2016 (UTC)[reply]
See Ritalin and cocaine. The trouble with saying "similar" is that, except in extreme similarity, it can be a bit arbitrary. I mean, on both structures you'll see a phenyl ring and a six-membered ring with five carbons and a nitrogen (piperidine) and an ester somewhere near where the two meet. But is that 'close enough to matter'? Usually it is more academically defensible to speak of the known activities of the drugs ... caveat being that these activities tend to be some of the most remarkably revised bits of scientific fact you can hope to run across, with different types of activity on different (usually newly discovered) receptors becoming the target of research from one decade to the next. I'm not sure there's any completely satisfactory way to look at this, however intuitive and meaningful the approach may be!
In terms of treating cocaine withdrawal with ritalin, I didn't see much - one thing I found was this abstract, which if I read it correctly is saying that ritalin actually increases cocaine cravings among those doing without it. I'm not sure I searched for this correctly however; there are some specialized terms about addiction that I cannot presently recall. Wnt (talk) 21:24, 19 March 2016 (UTC)[reply]
From a quick scan of the articles, they are both dopamine reuptake inhibitors. I wouldn't be surprised if the structural similarities you mention are the common cause of that, but I'm way out of my depth. --Trovatore (talk) 21:26, 19 March 2016 (UTC)[reply]
"Chemistry, Design, and Structure−Activity Relationship of Cocaine Antagonists". Chemical Reviews. 100 (3): 925–1024. 2000. doi:10.1021/cr9700538. has a section about methylphenidate analogues. DMacks (talk)

I've heard that evolution violates the second law of thermodynamics. Is that true? Bonupton (talk) 21:01, 19 March 2016 (UTC)[reply]

I heard it more generally that life does. That is, life is about increasing the order of the universe, which counters the general trend. I compare with the conservation of energy law, which states that energy can never be created or destroyed. That's not the full story, though, as mass can be converted to energy, according to E = mc2. StuRat (talk) 21:06, 19 March 2016 (UTC)[reply]
I linkified your title. StuRat (talk) 21:07, 19 March 2016 (UTC) [reply]
See Entropy and life and Objections to evolution for our articles on the subject. Tevildo (talk) 22:15, 19 March 2016 (UTC)[reply]
It might actually be that life and evolution are results of the second law of thermodynamics. ←Baseball Bugs What's up, Doc? carrots22:39, 19 March 2016 (UTC)[reply]
In short: No, evolution does not violate thermodynamics. You could substitute any production of apparent order for evolution - building a brick wall, or sorting a disordered drawer, would qualify too. In every case, the local or specific appearance of order is the result of processes which expel a lot of energy as heat, thus increasing the overall entropy of the universe. AlexTiefling (talk) 22:41, 19 March 2016 (UTC)[reply]
Note that both those examples are performed by life. There may be some non-life examples, though, such as the formation of crystals, which appear to be more orderly than the random arrangements they came from. Or does that imply that crystals, which can grow & reproduce, are alive ? Certain weather phenomenon, such as hurricanes, also seem more orderly than what pre-dated them, and the formation of a solar system or galaxy may qualify, too (and a galaxy would seem to qualify as a"closed system"). So, the tendency of everything towards a more disordered state seems like a rule of thumb, more than a law, to me. StuRat (talk) 23:22, 19 March 2016 (UTC)[reply]
That cup of coffee cooling on your desk is becoming more orderly by the second. Admittedly from your interest in domestic hygiene that may be due to living organisms, but to the engineer who has learned these things, a hygienic cup of coffee becomes more orderly but the surrounding atmosphere becomes less orderly at the same time, by a greater or equal amount.Greglocock (talk) 07:50, 20 March 2016 (UTC)[reply]
There's an interesting counter-example when artificial sweeteners are stirred into coffee. Since they are much more powerful, they frequently use silicon dioxide (quartz sand powder) as filler. It doesn't dissolve in the coffee, but rather stays in suspension for some time after the stirring motion. If left to sit, however, it settles out on the bottom. So, in this case, life (the person who stirred it) made it less orderly (mixed), and time, without the requirement for life, allows it to become more orderly (separated). I suppose evaporation also tends to make things more orderly, as the residue is then separated out.StuRat (talk) 19:09, 20 March 2016 (UTC)[reply]
By that reckoning, just about any mechanistic machinery that we have observed, such as the dampening of string vibrations or the eventual dampening down of a playground swing set to its "ground state", or the buildup of layers of sediments, and chemical analogs such as crystallization would also be "counterexamples" forth with that have simply gone unnoticed. <sarcasm intended> No Stu, :), if we are together to ever evolve to the point of a counterexample it will be in spite of it all. It takes considerably more comprehension, observation and ingenuity to arrive at such a point however.. health permitting. --Modocc (talk) 19:48, 20 March 2016 (UTC)[reply]
My counter-examples are not meant to disprove the 2nd L of T, just show that it doesn't apply at every level. StuRat (talk) 21:28, 20 March 2016 (UTC)[reply]
Yeah, nice try - but your examples are clearly incorrect. Evaporation decreases the entropy of the material left behind but vastly INCREASES the chaos in the material that evaporates off and gets mixed into the air currents - overall, the entropy of the system increased. When a string is dampened down, you may argue that it became more 'organized' - but the otherwise calm air around it (that was responsible for the dampening) was stirred up and made more chaotic - and the well organized, constrained elastic energy that was causing the vibration was dissipated as low-grade heat. The entropy of the string decreased (arguably) - but the resulting motion of the air and the low grade heat that was produced were a massive increase in entropy. Stirring your coffee imparts a very organized kinetic energy to the liquid - which gradually slows down as a result of turbulence and friction to allow the sediments to settle out. All of that organized kinetic energy in the liquid's rotation was dissipated as low-grade heat - so, again, entropy increased. With EVERY example and at EVERY 'level', if you look carefully enough, you'll find that the overall entropy increases...no exceptions whatever. If you think you found an example where it decreased, then you haven't thought hard enough about ALL of the consequences. 15:24, 23 March 2016 (UTC)SteveBaker (talk)
But the 2nd law does apply at every known level (unless you have designed and perhaps operated a perpetual motion machine of the second kind or perhaps considered why relativity is wrong :-)), for the law applies to all known spontaneous processes. --Modocc (talk) 21:40, 20 March 2016 (UTC)[reply]
Relativity is "wrong", or at least insufficient to explain the universe at all levels, hence the need for quantum mechanics. StuRat (talk) 06:16, 21 March 2016 (UTC)[reply]
  • To explain briefly why the statement is wrong: the second law of thermodynamics only applies to closed system -- that is, systems that don't interact with anything outside themselves. The Earth is not a closed system, because it receives a constant influx of energy from the Sun. Without some such energy source life (and evolution) would be impossible. Looie496 (talk) 22:47, 19 March 2016 (UTC)[reply]
Exactly. Either the literalists don't have a clue about science, or they do understand it just well enough to think that they understand it and that it supports them. The second group are more of a problem, because they can be seen incorrectly as scientific. Robert McClenon (talk) 02:42, 20 March 2016 (UTC)[reply]
The second law of thermodynamics is the only physical law that isn't time-reversal invariant (aside from the tiny T-symmetry violation in the Standard Model). That means that the second law is implicated in anything that happens in one direction of time and not the other. Evolution is one of those things. The ratchet effect that leads to organisms becoming better adapted to their environment over time (rather than randomly better or worse averaging to no change) is a manifestation of the second law. So it's wrong to say that evolution violates the second law, but also wrong to say (as people sometimes do in response) that it evades the second law (e.g. because the law only applies to closed systems). On the contrary, evolution is an example of the (nonequilibrium) second law in action. -- BenRG (talk) 00:18, 20 March 2016 (UTC)[reply]
@BenRG: There are interesting things though when Hubble constant and expansion are not time invariant and speed up and slow down. The same process may not yield the same heat or entropy, which strictly speaking is a violation. Variable rates associated with dark energy and dark matter may very well mess with our understanding of how the entropy of the universe changes and whether we "borrow" or "lend" entropy to other epochs. --DHeyward (talk) 04:18, 24 March 2016 (UTC)[reply]
Usually when I hear this come up, they are using an oversimplified statement of the law. I think it applies to a closed system, and the Earth is not a closed system. See this from the article:

The entropy of the universe tends to a maximum. This statement is the best-known phrasing of the second law. Because of the looseness of its language, e.g. universe, as well as lack of specific conditions, e.g. open, closed, or isolated, many people take this simple statement to mean that the second law of thermodynamics applies virtually to every subject imaginable. This, of course, is not true; this statement is only a simplified version of a more extended and precise description.

Bubba73 You talkin' to me? 00:53, 20 March 2016 (UTC)[reply]
The second law does apply to virtually every subject imaginable. It's why water flows downhill and not uphill. It's why evolution (a hill climbing process) makes progress up the hill. It's why you remember the past and not the future. It's why the sun shines. Every process that happens more often than its time reversal is that way because of the second law. It's so pervasive that people tend not to recognize that it's there. -- BenRG (talk) 08:58, 20 March 2016 (UTC)[reply]

Any statement about the Second Law of Thermodynamics made by anyone other than an academic physicist should be assumed incorrect, absent strong evidence to the contrary. Shock Brigade Harvester Boris (talk) 02:19, 20 March 2016 (UTC)[reply]

I will just leave this here... [5][6] --Guy Macon (talk) 05:32, 20 March 2016 (UTC)[reply]

Yes, viruses evolve enough each year so that we don't have resistance (and last year's flu shot is made ineffective.) Bubba73 You talkin' to me? 05:40, 20 March 2016 (UTC)[reply]


With all of these kinds of law, you need to draw a box around a "system" and measure the property inside the system - taking careful account of what enters and exits the system. Choosing the right box can make this either fiendishly difficult or ridiculously easy.
I don't know why people find this so difficult - it's no different than any of the conservation laws. How is it that our 3lb puppy grew to an 80lb dog? Was the law of conservation of mass violated? Well, no - if we total the amount of puppy chow, water and oxygen consumed and the amount of pee, pooh and CO2 produced, we'd find that there was a continual slight discrepancy between what went into the puppy and what came out - which exactly balances the weight growth of the dog.
It's not just evolution that superficially appears to drive entropy backwards - life in general seems to do so even when it's not evolving. But that's an illusion that comes about from picking the wrong box. A puppy consumes high energy food and poops out low energy waste, along with a lot of low-grade heat. The entropy of the dog decreases - but at the cost of a much larger increase in entropy of the world around it. Entropy is at least conserved - and actually increases overall...just as the second law predicts.
This also applies to the things lifeforms do. If I decide to spend the afternoon sorting my DVD collection into alphabetical order - then I'm decreasing the entropy of my DVD collection - but in doing so, I consume more calories than if I'd sat in a chair and done nothing. So in the process, I ate an extra cheese sandwich - and turned more low entropy cheese sandwich into high entropy waste heat to counteract the (marginally) decreased entropy in my DVD collection.
We see this everywhere - whenever humans build something (locally decreasing entropy in the process) we trash the planet to a greater degree - increasing entropy all around us.
Evolution is just another process that life engages in. In the course of evolving (which we presume to be a largely reverse-entropy process) living things produce more disorganized waste than they otherwise would...so the entire process drives entropy up as the second law predicts.
This really isn't difficult.
SteveBaker (talk) 14:29, 20 March 2016 (UTC)[reply]
Life doesn't really increase the order of the universe. Nor does non-life avoid increasing in order locally. A volcanic eruption, all that heat in one place, crashing steaming into the sea - lots of free energy there. That doesn't disprove the second law, but just goes to show that so long as you have an energy source of some kind, interesting things keep happening. Wnt (talk) 17:36, 21 March 2016 (UTC)[reply]

I thought about this. Crystals form on their own and they are highly ordered. If the anti-evolutionist misinterpretation of the laws of thermodynamics were correct, crystals would not form. But they do - it is a common grade school science experiment to grow crystals. Bubba73 You talkin' to me? 18:52, 21 March 2016 (UTC)[reply]

Perhaps instead of just thinking about it and then jumping to a conclusion people should read Crystallization#Thermodynamic_view? The laws of thermodynamics are probably the most solidly proven laws in all of science. There are no known exceptions - and if an exception were ever to be demonstrated, it wouldn't be something so well studied as evolution or crystal growth. Established laws are sometimes (very rarely) overturned - for example when the laws of "conservation of mass" and "conservation of energy" were found to be violated in nuclear fission/fusion and had to be replaced with the law of "conservation of mass/energy". But this has to happen in areas that are not already well-supported by scientific evidence - generally they require experiments that are out on the extreme edges of what is known. Crystal formation and evolution are both very well studied - if they somehow violated thermodynamics, we'd know about it for sure and we wouldn't be teaching them as "Laws of Nature" anymore...just as we don't teach "Conservation of mass" anymore. SteveBaker (talk) 19:58, 21 March 2016 (UTC)[reply]

Actually I would think the various acceleration/deceleration accounts of the universe are more likely to violate the 2nd law of thermodynamics. I doubt "increasing entropy" is any more golden than Newton's law and we will discover more about why we generally observe increased entropy in everyday life. But dark energy, dark matter, variable rates of expansion and all sorts of cosmological weirdness will simply redefine it. But the second law of thermodynamics regarding life isn't a violation unless the sun is doing something we don't know. It's macro, not micro. Throwing a ball in the air doesn't defy gravity either. --DHeyward (talk) 03:10, 24 March 2016 (UTC)[reply]

March 20

Null result of Michelson–Morley experiment extrapolation

Question Remark
(I was a bit hasty.) Why does Feynman say so steady that Michelson–Morley experiment has null result? Next from this statement he deduces a Lorentz formula for . But experiment guarantees null interference displacement only for observer in interferometer reference frame. No guarantee that observer at rest (is not moved with Earth) also will see null result. To check this latter observer must run with 1/4 Earth speed , which is impossible.

So can anybody explain me, why must any observer see null result? Maybe moving observer does not see null result. If so, we can detect absolute motion, or what? But how? It just will show that moving observer has speed relative to interferometer. 95.134.204.112 (talk) 05:04, 20 March 2016 (UTC)[reply]

http://www.feynmanlectures.caltech.edu/I_15.html#Ch15-S5, "yet the null result of the experiment demands that the times must be equal"
So we have 2 observers: 1st over interferometer and 2nd on Sun. 1st observer sees null result as should. When he switches on interferometer, he sees spherical wave with epicentre in lamp and this epicentre remains in lamp. Wave reaches both mirrors at same time and then comes back to screen at same time giving inreference.

2nd observer sees spherical wave of light with epicentre in lamp but this epicentre is fixed, has speed in his reference frame associated with Sun, so lamp with speed is immediately mismatching epicentre. So as with ether concept, wave front reaches upper mirror in . He sees also that 1st observer's watch shows time as arm BC of interferometer can be considered as primitive clock, ticking when ligth signal reaches mirror.

Time needed spherical wave to reach right mirror is . Time backward is . So like with ether concept total time . But Feynman for unknown reasons concludes that 2nd observer sees undisplaced interference. That needs Lorentz contraction. Q.E.D.

But it's ridiculous! Feynman must first prove 2nd observer sees undisplaced interference as 2nd observer is phantom.


95.134.204.112 (talk) 09:39, 20 March 2016 (UTC)[reply]

http://www.feynmanlectures.caltech.edu/I_15.html#Ch15-F2
An experiment only has one outcome. If the interference fringes are projected on a screen, then as the apparatus is rotated the fringes will either shift or not. The motion of the person looking at the screen doesn't matter. (This was something that Einstein understood in his original exposition of relativity; he clearly distinguished observers (who were scientists making observations) from the things they were observing (such as the positions of the hands of clocks). He talked about, for example, "an observer who takes S as his reference frame", meaning a scientist who records times shown on the clocks labeled S when they are coincident with other objects. The scientist's motion is unspecified and irrelevant. Later expositors conflated observers and reference frames, which led to the confusing modern sense of "observer" in special relativity. When you look at an experiment while moving rapidly past it, you are an observer in Einstein's sense, and your reference body is the experiment.) -- BenRG (talk) 08:15, 20 March 2016 (UTC)[reply]
I don't understand. So any observer is flying with interferometer with closed eyes. When he opens eyes, he stops for a millisecond. It changes nothing. When he stops, he cannot see interference as interferometer moves with speed .
And we cannot imitate such observation (because cannot reach speed 1/4 of Earth). So such observation is completely theoretical, besides unproved. We cannot derive other statements from this unproved.95.134.204.112 (talk) 10:58, 20 March 2016 (UTC)[reply]
It's an assumption of every physical theory that different people see the same readouts on screens. Screens emit light (even projection screens, by scattering it). That light propagates in all directions and some of it eventually hits the retina of someone's eye. If the screen is showing the number 42, the light encodes those digits and anyone who looks at the screen will see those digits. They may be distorted by aberration, but aberration can't turn Arabic numerals into different Arabic numerals. The same is true of M–M fringes. If you draw lines on the screen where the peaks of the fringes are, then rotate the apparatus, the peaks can't shift relative to the lines for one person and not shift for another. I don't know that it's logically impossible for a radically subjective world like that to exist, but we don't live in it.
The objects you see (via light transmitted through vacuum) at any given moment are on your past light cone. The direction of the light cone is determined by the spacetime geometry. There are different light cones for different points of spacetime but not for different states of motion. So the observer will see the same thing on the screen (modulo aberration and redshift) whether or not he stops. -- BenRG (talk) 17:39, 20 March 2016 (UTC)[reply]
If you are moving with the interferometer, it is in your frame reference. Michelson-Morley would not be able to detect relativistic differences between themselves and the interferometer. And they didn't. Someone passing by Michelson-Morley's apparatus near the speed of light would not describe the apparatus as having equal light path distances. They would describe an apparatus with different length paths. Lorentz made that up. Einstein proved it. Two observers in different frames of references though, will measure the speed of light as being the same and nature will conspire to perturb time, length, and mass to make it the same. The next tricky part is rotating the Michelson-Morley interferometer as someone passes by. Oops, we no long have an inertial frame but an accelerating frame of reference. If the observer is moving fast as it's rotating, it moves into a GR category. Then you have all sorts of weird observer effects. --DHeyward (talk) 05:16, 24 March 2016 (UTC)[reply]
I imagine you have to do some fancy footwork with relativity to make sense out of this. The point isn't really that the numbers on the screen don't change, but that the light really gets the same distance at the same time in any frame of reference. Even though the distance is changing with the Lorentz contraction and the time is subject to time dilation! And the number of wavelengths in that stream of light have to remain the same per redshift/blueshift so that you can count up just as many nodes in the pattern wherever you look at it from. It all has to work out so if you mark out a meter, the light goes exactly that meter in the same time as any other light goes any other meter in that frame of reference (in vacuum, at least) ... no matter what. But I am not feeling so ambitious as to write out all the mathematics just now. Wnt (talk) 17:44, 21 March 2016 (UTC)[reply]
@Wnt: Ahh, but Michelson Morley experiments show no red shift or blueshift even though they use the velocity of the earth in one direction and orthogonal paths. The key is that the observer is not moving relative to the apparatus. They measure no dilations of any sort. Only an observer moving relative to the apparatus would see that but they wouldn't describe it as Michelson Morely did. It would look quite different. We can tell relative motion only, not absolute so all SR effects are measured against other moving frames. Anything in our inertial frame is constant. --DHeyward (talk) 05:01, 24 March 2016 (UTC)[reply]
Michelson-Morley experiment was a hypothesis regarding the medium in which electromagnetic waves traveled. We know now that the speed of light in free space is constant. However, it is not constant in dielectric mediums. It's slower. Always. Michelson-Morley hypothesized that free space was just another dielectric medium which we all move through. Light through a prism shows the effect. They expected to be able to observe our relative movement through this medium. After all, if it was static, an orthogonal apparatus might look like 4 inches of glass in one direction and 2 inches in the orthogonal direction and repeating the experiment 3 months later or simply rotating the apparatus should show our direction. They failed to detect any medium and light was the same in every direction. This was confusing since they already knew about dielectric propagation (Maxwells equations basically include free space as a dielectric medium) and failing to find a dielectric of any sort wasn't expected. The null result is the "null hypothesis" as that was stated within the margin of error, I don't think it's a fringing null (could be wrong as fringing would be the same in in orthogonal directions and Feynman might be having a go at a double entendre. Had they been correct, they could demonstrate the speed of light varied with our relative motion in the dielectric. What they didn't know was how fast the solar system was moving and whether the speed of the earth's motion around the sun was significant. So they kept trying with more and more sensitive apparatus with the assumption the change in earth velocity was too small. Sometimes your life's work is only a warning to others. Einsteins photoelectric effect, not relativity, probably threw a bigger monkey wrench in the wave propagation theories when "particle" became part of the theory. --DHeyward (talk) 04:47, 24 March 2016 (UTC)[reply]
the Feynman answer lies in that the observer in Michelson morley is not moving relative to the apparatus and sees no difference - no red shift, no blueshift, no speed difference. Zip. There is no absolute frame of reference, only observer relative motion. --DHeyward (talk) 05:04, 24 March 2016 (UTC)[reply]

Dandelion wine

Is it possible to literally make wine (or other fermented beverages) from dandelions? 2601:646:8E01:515D:2D19:E107:8069:6081 (talk) 10:24, 20 March 2016 (UTC)[reply]

Not without any other ingredients, but it certainly is possible to use them as the main flavour ingredient: see here for example.--Phil Holmes (talk) 11:34, 20 March 2016 (UTC)[reply]
To make any wine you must have some sugar for the yeasts to convert into alcohol. All plants contain some sugars, and flowers contain some as nectar, so there must be some sugar in dandelion flowers. I suspect the wine would be pretty weak if you didn't add more sugar to the mix. By the way, I wouldn't bother trying. My grandmother used to make it - absolutely disgusting! 109.150.174.93 (talk) 11:59, 20 March 2016 (UTC)[reply]
Likewise. To make wine one needs fermentable sugars that can be turned in to alcohol. Dandelions (and many other flavouring ingredients in wines) don't have this but they give their name to them because they are the main flavourant. Just as absinthe and ouzo did not obtain any of the alcohol from wormwood nor the flavourings use in ouzo. Just as sloe gin uses sloes only as a flavourant, in-contrast to malt whisky that actually uses malted barley as a source for the fermentable sugars. So, for Dandelion Wines etc., ingredients with fermentable sugars need to be added.--Aspro (talk) 12:36, 20 March 2016 (UTC)[reply]
As you can see from this recipe, the bulk of the fermentable material is derived from grapes in various forms, with the dandelion heads there as an adjunct. --TammyMoet (talk) 13:34, 20 March 2016 (UTC)[reply]
Since just about any plant matter will ferment, I suppose you could make alcohol out of dandelions alone (hopefully they would get the yeast from the air), but you would need lots of dandelions and would get very little alcohol. Also, it would likely taste horrid. StuRat (talk) 19:03, 20 March 2016 (UTC)[reply]
So I take all this to mean: YES for dandelion-flavored wine, and NO for wine from dandelions alone? Thanks, everyone! 2601:646:8E01:515D:E082:2C68:8C6B:47A9 (talk) 23:24, 20 March 2016 (UTC)[reply]
As for fermented beverages that are not wine, see dandelion and burdock which (in its traditional form) is a root beer. The extra sugar would traditionally be black treacle (AKA molasses) or golden syrup. You normally include more burdock than dandelion (this recipe uses 150 g burdock root to 50 g dandelion root), but I suppose you could use dandelion root alone - but these are very bitter. Smurrayinchester 09:50, 21 March 2016 (UTC)[reply]

cultural acceptance of homosexuality lead to its evolutionary demise??

if homosexuality is maladaptive as far as evolutionary theory (as article suggests..where the only criteria for success is passing on one's genes) then will it die off completely if homosexual lifestyle becomes fully culturally accepted...have the genes continued primarily because homosexuals felt pressure culturally to marry/have heterosexual sex relationships, thereby passing on their genes???? the articles suggests science views it as genetic...and that science is trying to figure if some 'homosexual genes' are beneficial to heterosexual reproductive success...but even if these genes continued it wouldn't mean the continued existence of homosexuals but the continued existence of heterosexuals with a partial amount of so called 'homosexual genes.'...so could there theoretically be a period of full homosexual acceptance worldwide that lasts a couple centuries and then they disappear from the earth altogether.....or perhaps more interesting...is it discrimination itself that the existence of homosexuality owes itself to...????68.48.241.158 (talk) 13:14, 20 March 2016 (UTC)[reply]

Please see Homosexuality - Evolutionary perspectives. The Ref. Desk will not provide opinion, prediction or debate beyond the studies cited in the linked article. AllBestFaith (talk) 13:50, 20 March 2016 (UTC)[reply]
yeah, I read that and 'homosexuality in animals' article (which says there are probably no exclusively homosexual animals)..that's what led to my question...my question is about genetics...is it possible that the genes that lead to "full homosexuality" or an exclusive homosexual lifestyle can continue on if these people are never pressured into heterosexual reproduction, and therefore never participate in it..?? perhaps they would still pop up, but just much more rarely?? 68.48.241.158 (talk) 14:04, 20 March 2016 (UTC)[reply]


Certainly. If you pick a less emotionally charged human trait (and Sickle cell anemia is my 'go to example' here) - we find an entirely genetic disease - that (without treatment) would be fatal before the age at which reproduction is likely. Why didn't this gene vanish from the population? In fact, in certain populations, it's a very common genetic disease. Well, it turns that you only get the disease if you inherit two copies of the gene. But if you inherit just one copy, you become much more resistant to malaria than people who don't have the gene at all. So people with one copy of the gene who live in areas where malaria is common have a pro-/con- situation here. The presence of one copy of the gene decreases the risk of death due to malaria in early childhood so if you have one copy of the gene, more of your children survive malaria - but if your spouse also has the gene then some of your children may get two copies of the gene and die of anemia. Evidently (since the gene is still around), the benefits of fewer malaria deaths exceeds the cost of more anemia deaths - so evolution favors keeping it around. This leaves us with an interesting situation, where if only a few people have a single copy of the gene - then the odds of two people with the gene meeting and having children is very small - and children of a person with one copy are about 50/50 likely to inherit it and get the benefit. But if the gene were to take over the entire population, then one child in four would die of anemia with only half of children still getting the anti-malaria benefit. So the system self-regulates to a point where between 10% and 40% of the population carries a single copy of the gene - with the percentage being higher where malaria is most common.
So this proves that not all genetic conditions are selected out of the gene pool by evolution - and not all of them take over the entire population. Some have to produce a delicate balance between enough copies of the gene to get the benefit - without sufficient copies to produce a downside.
The thing about homosexuality is that it doesn't seem to depend on just one single gene - so it's not so easy to pinpoint. (Although there does seem, to be statistical evidence for a genetic component). Therefore, if being homosexual is akin to having two copies of the sickle-cell gene (in terms of reduced chances of reproductive success) - then the gene (or more likely, genes-plural) must be increasing the reproductive success of the homosexual person's parents or their siblings in some manner that exceeds the risk of having a homosexual child and therefore no grandchildren.
When a single gene is involved (as with sickle cell anemia) it's rather easy for geneticists to figure out what that benefit is. But when there are multiple genes involved, it can be exceedingly difficult to understand the interactions between them - so we very often don't know what the benefits are - but we can be sure that they must be there, or (as our OP suspects) the genes would be selected against and eliminated from the gene pool in just a few generations.
It's very tempting to look at the stereotypical traits of gay people and wonder whether those traits would be present and valuable in their parents - but it's a very tricky matter. There is no obvious trait that you'd see by examining sickle-cell anemia victims that would predict the presence of the gene in their parents...and that's a much simpler genetic situation than in homosexuality.
Knowing these things, whether cultural acceptance would make a difference is hard to say. I suppose that cultural pressure to hide ones' homosexuality forces some gay people into heterosexual marriage and a non-zero probability of having children - which would allow these genes to propagate more rapidly than if cultural acceptance had made those marriages less likely. But that could only adjust the balance because even a reduced (but non-zero) chance of reproducing would apply sufficient evolutionary pressure to eliminate the gene after sufficient time has passed. Given enough time, evolution can eliminate genes if there is only a tiny downside to them.
So, on balance, I doubt that cultural acceptance can make any significant dent in the matter either way. Far more likely is that there are subtle reproductive benefits to people who only have some fraction of the genes involved - and that would continue to maintain the genes in the gene-pool, no matter the cultural changes around homosexuality.
SteveBaker (talk) 15:28, 20 March 2016 (UTC)[reply]
I'm puzzled as to why the OP feels that gay people wouldn't want to have children. I pretty sure that gay people feel the same way about having kids as non-gays. CambridgeBayWeather, Uqaqtuq (talk), Sunasuttuq 00:37, 21 March 2016 (UTC)[reply]
There are many, many gay people who have children of their own and/or adopt. ←Baseball Bugs What's up, Doc? carrots00:47, 21 March 2016 (UTC)[reply]
I believe that the OP's assumption is that "out" gay people have fewer children who carry their genes, and thus are less likely to pass on those genes. StuRat (talk) 06:00, 21 March 2016 (UTC)[reply]
Another page to check out: Biology_and_sexual_orientation. That said, it's a big assumption to assume sexual orientation is the result of heritable genes or alleles. Epigenetics, neurobiology, hormones, etc. have all been suggested as possible casual factors. Further, it is incorrect to assume something must be genetically beneficial to survive natural selection. Rather, a heritable trait must either be selected for (beneficial) or selected against (detrimental), but it's possible to have neutral and vestigial traits. EvergreenFir (talk) Please {{re}} 06:07, 21 March 2016 (UTC)[reply]
As far as homosexual genes being beneficial to reproductive success, here we might look at bonobos, which engage in all kinds of sex, including homosexuality. There it seems to serve a social purpose, by reducing tensions and making war less common than in chimps, thus helping all to survive and pass on their genes. I don't know if any bonobos are exclusively homosexual, but, if so, that could be explained by Steve's sickle-cell example above, or by helping the whole group to survive, so that bonobos related to them pass on their genes more successfully, even if they themselves don't. StuRat (talk) 06:06, 21 March 2016 (UTC)[reply]
it seems then that if there's a requisite genetic component required (even if some environmental requirement too)...then the theoretical full acceptance of homosexual lifestyle over the centuries would have hugely reduced the homosexual population that exists today...perhaps possibly from, say 10% of population, to perhaps less than 1% of population...(obviously the point above about adoption is irrelevant and missing the point...but homosexuals passing on their genes with surrogates or with new technology going forward would be relevant)..but can't find anything along these lines in literature (ie discrimination itself being the mechanism by which the thing discriminated against exists...seems like something an academic could make a reputation on)..but perhaps would be considered way politically incorrect within academia as stating that homosexuality is technically maladaptive within evolutionary theory is probably itself a bit of a no no (I read an article once that suggested the high rate of homosexuality in the priesthood is due to the socially accepted cover it allowed men who had no desire to marry/have heterosexual sex relations.....) 68.48.241.158 (talk) 12:39, 21 March 2016 (UTC)[reply]
Hypothetically, it could be that there isn't an allele for "homosexuality", but rather an allele for "wanting to have sex with men". Whether that was evolutionarily maladaptive would depend on who inherited it. Iapetus (talk) 13:41, 21 March 2016 (UTC)[reply]
The argument about social pressure makes perfect sense, except ... other mammals have comparable rates of homosexuality. I am not aware of any explanation of homosexuality rates that makes any sense at all. Even the fecundity argument falls short, because I can't believe a biological system can't adapt to work differently in men and women, given how many major organs do. Wnt (talk) 11:46, 23 March 2016 (UTC)[reply]
There are plenty of cases where genes that are advantageous for women but are disadvantages for men (and vice-versa). The genes that produce different kinds of green receptors in the eyes of hunter-gatherer women would allow a group of them to better sort ripe fruit that they've gathered from unripe fruit. But those genes results in a significant proporition of the the males being red-green colorblind...which isn't great for them - but probably doesn't reduce their hunting skills. That's great for hunter-gatherers - but fails badly when we're all office workers.
Since half of our genes come from a woman and the other half from a man, it makes sense that some of these things will wind up being compromises in which one sex gets a statistical advantage and the other a disadvantage. Ideally, these useful-to-one-sex-only things get confined to the X and Y chromosomes - but useful mutations may not arrive there so conveniently. In such cases, evolution will very effectively do a hard-nosed statistical calculation of cost vs. benefit and keep or reject those genes based on the overall reproductive rate of the group. So it's entirely possible for severely disadvantageous genes to remain in the gene pool if they have a small 'upside' for a large enough fraction of all people. My example of the sickle-cell gene is a classic example of that - even if both parents have the gene, having half of your kids not die of malaria trumps a quarter of them dying of anemia - but only in an area where malaria is very common. The statistical distribution of those genes follows the places where malaria is (or recently was) common with extreme precision. SteveBaker (talk) 15:10, 23 March 2016 (UTC)[reply]
"other mammals have comparable rates of homosexuality..." that's just it...I don't think that's true...apparently exclusively homosexual preference doesn't exist in the animal kingdom...and the primary thing that distinguishes humans from animals is advanced social constructs/pressures...thereby leading (paradoxically) to homosexuality's continuation (ie it is selected out of existence by all lower animals...) possible??68.48.241.158 (talk) 18:33, 23 March 2016 (UTC)[reply]
Hmmm, I see there's also some self-contradiction in our article Homosexual behavior in animals which says it is rare, but later says it is common in a variety of other species. I haven't looked into this closely but we probably ought to nail that down better. Still, a high rate in any non-human species pretty well rules out social compulsion - I mean, sheep may be notorious followers, yet I doubt that a hundred years ago they had draconian punishments for gay rams who had to learn to play along at being straight to get by. I mean, that seems like pretty deep thought for a sheep... or perhaps they are just smarter than to make such nonsense rules to begin with. Wnt (talk) 00:27, 24 March 2016 (UTC)[reply]
Sounds like it's rare, in general, but common in some other species. Heck, lesbian cows were even useful in determining which other cows were season (they would attempt to mount them). StuRat (talk) 02:16, 24 March 2016 (UTC)[reply]
it's discussed in the article but nonetheless seems labeling animals 'homosexual' is extremely misleading in and of itself....it mentions same-sex sex behavior as part of the animals overall repotoire, this makes sense, I suppose...as more or less keeping in practice for reproductive sex among other things...I don't think exclusive homosexual attraction/interest can be observed in the wild (but how would such even be observed??)..though perhaps it exists as a very, very rare anomaly...and it would perhaps exist in humans only as a very, very rare anomaly as well but for social pressures....)...anyway..68.48.241.158 (talk) 16:14, 24 March 2016 (UTC)[reply]

Silver polish

Grandma's silverware stored away is dark, tarnished and looks like junk. But when she about to host a formal dinner, she got to work with some stuff in a bottle. After a while, the silverware looked like new again and was white and shiny. Within a few months it turns dark and ugly again. What's the stuff in the bottle? It seems you can't buy this silver polish in local shops anymore. Only things based on pumice power or other abrasives designed to clean saucepans. — Preceding unsigned comment added by 120.145.169.202 (talk) 13:50, 20 March 2016 (UTC)[reply]

Well, this is 2016. If you can't buy something locally - then search online. I typed "Buy silver polish" into Google and got a bunch of supermarket links for a half dozen brands - some of which I could get locally. Also links to people selling the stuff on Amazon. If you know the particular brand your grandma uses - just Google it with the phrase "buy XXX silver polish" - where "XXX" is whatever she swears by. Since these kinds of searches use your location information - you'll need to do it for yourself. SteveBaker (talk) 14:02, 20 March 2016 (UTC)[reply]
Tons of non-abrasive silver polishes, including some which deposit a thin layer of silver on the surface, and some which chemically remove the sulfur which causes the tarnish. Urea salts use to be commonly used to "dip" silver collectible coins, for example. For household silver, Wright's silver polish seems to be a pretty universal choice (not a commercial recommendation here). For keeping tarnish from re-occurring, and to avoid fingermarks, you might try a microcrystalline wax (used by the British Museum on almost everything from paper and coins to paintings and furniture). Collect (talk) 14:05, 20 March 2016 (UTC)Collect (talk) 15:02, 20 March 2016 (UTC)[reply]
I don't know what urea salts do to silver coins, but the rule generally is never to polish collectible coins. They acquire a valuable "patina" which is removed by the chemicals in the polish. 2.221.197.64 (talk) 14:33, 20 March 2016 (UTC)[reply]
"Toning" is, at times, and for modern collectible coins, sometimes quite ugly. While grading services try to mark cleaned coins as "cleaned", a coin which then acquires toning (say, by placing in contact with paper containing sulfites for a period of time) is generally more easy to sell than one with "ugly toning". The urea dip and distilled water rinse does not "polish" the coin, by the way. PCGS states: No matter how the results are achieved, many coins have been improved by judicious cleaning with commercial dips, solvents, or plain soap and water. PCGS grades many coins that have had their surfaces altered by the removal of "problems," perceived or otherwise. The coins it does not grade are the ones altered by adding substances to the surface or altering the surfaces by physical methods. Collect (talk) 15:02, 20 March 2016 (UTC)[reply]
The key word here is "collectible". Sure, if your goal is to eventually sell the item as a valuable piece of antique silver to a fanatical collector - then not cleaning it is probably a good idea. However, that's not everyone's goal. If I wish to actually use grandma's silver tea set for serving tea - then nobody is going to be very happy with a grungy, blackened milk jug. So you have to clean it - even if doing so will gradually wear away the sharp relief of the design and make it less attractive in the event you ever wanted to sell it. You don't HAVE to treat antiques as things to be revered, that have to look ugly and be entirely non-functional in order to preserve their value - it's equally valid to use end enjoy them as they were intended to be used so long as you understand that they may depreciate in value as a result.
SteveBaker (talk) 14:44, 20 March 2016 (UTC)[reply]
That’s almost a good point. But the OP states a formal dinner suggesting all the right accoutrements but you mention a silver tea set for serving tea. Silver-tea-sets were once very common and popular amongst the more lower middle classes in Britain. They where made from Sheffield plate. A very, very thin (and thus cheap) coating of pure silver that was fused to the base alloy – and the very very thin pure silver layer doesn't turn black so readily. Yet, with only a few decades of polishing it wears away and reviles the base metal. Which is why one only sees them now on things like the Antiques Roadshow were the owner has had locked away in a cardboard an never been used since he inherited it from his greate-late-maided- aunt or whomever. With a valuation put on it that wont even make dent in his daughter's credit card deficit . Most silver tea set stuff is lower middle class crap. Yet, the OP's states that s/he Grandmother uses her silverware. The OP's grandmother sound to me as though she has the real McCoy – the solid good quality stuff. Which both needs taken care of but enjoyed during our short sojourn in life. --Aspro (talk) 15:49, 20 March 2016 (UTC)[reply]
Note my earlier point - that there are polishes which deposit a thin layer of silver, which is what a lot of "antique dealers" do for thin spots on silver-plate <g>. Further - some patterns may, in fact, be highly sought after, mirabile dictu. Collect (talk) 18:01, 20 March 2016 (UTC)[reply]
My mother swears by baking soda mixed into a paste. You can also use toothpaste. Another recipe is to put the silver into a bit of aluminium foil in a bowl, add salt and baking soda to the bottom of the bowl, pour on white vinegar to disolve the soda and then cover in boiling water. Leave until cool enough to remove with your fingers and polish with a soft cloth. --TrogWoolley (talk) 14:35, 20 March 2016 (UTC)[reply]
Toothpaste in general is far too abrasive for silver - it is used, however, for polishing acrylics. Collect (talk) 15:02, 20 March 2016 (UTC)[reply]
Oh. Don't know where to start on this. Low grade silver will tarnish very quickly and badly but it may be you grandmother is storing it where the is high humidity (i.e., above 40%), (which will occur in more parts of the world). Placing them (after being cleaned) in a sealed plastic bag with some silica gel, may leave the silverware requiring nothing more than a final soft-cloth polish for those special occoations. Remember, every time one cleans silverware it wears some of it away... and lowers it value. Silver polish and aluminium/backing soda is still on sale everywhere. Just google it. If one-day you are hoping to inherit her cutlery, it may be worth genning up on the expert views. Suggest one starts here: Victoria and Albert Museum Kensington, London, Great Britain. Also, the silverware may be hallmarked, so from that you can find the quality and ascertain if it just gets tarnished from simply little use and bad storage. You may save your Gran quite some time (-especially if it is a silver-service of many forks, knives, spoons, tureens etc.,) in polishing with needles silver polish, when all it requires is a soft-cloth to bring up the lustre if properly stored.--Aspro (talk) 14:38, 20 March 2016 (UTC)[reply]
Re: V&A : Microcrystalline wax is used there routinely - but they also do have a special cellulose nitrate lacquer for silver if tarnishing remains a problem. Collect (talk) 18:09, 20 March 2016 (UTC)[reply]
Hi 120.145.169.202,
After all the advice above you have to use your common sense. If Grandma was keeping these items in a glass display case for view only, then I would say use microcrystalline wax but that is over doing it. For one, the best stuff is a sod to wash off. Hot water and washing up liquid wont do it. Second, small imperfections may cause pitting at that point (i.e., creates a electrolytic cell) which can be created by use. Lacquer, forget it, if the utensils are being used for the purpose they were designed for - pit corrosion again. Now that we (you perhaps too) don't rely on gas lighting from coal gas, there is little sulfur in the air other that from diesel vehicles. So, I would go for the simple practical solution of keeping the sulphur out by purchasing a large enough polythene bag to hold them all (including the baize lined case if they are of top quality and are contained in one). Include a silica gel sachet or two to keep out the moister (get a sachet that has a dye indicator to show when it is exhausted), (the dyes may be carcinogenic, but if you do not eat them you'll be OK). If you must, included a sachet or two of an oxygen scavenger to prevent oxidization. Doing more (in my view) is a complete over kill. Finally: This time next year I may be in Western Australia – can I pop round for dinner and I promise to do the washing up afterwards and take out the trash.--Aspro (talk) 19:39, 20 March 2016 (UTC)[reply]

Isn't there a clever trick for cleaning silver using electrolysis, which gets the tarnish to hop onto a piece of silver foil? --Dweller (talk) Become old fashioned! 13:01, 21 March 2016 (UTC)[reply]

BTW, we have an article on Microcrystalline wax. -- 2600:1004:B016:7C7A:5F9:69FF:E30A:CDED (talk) 16:31, 22 March 2016 (UTC)[reply]

Pacific Ocean disappearing

When is the Pacific Ocean expected to disappear? — Preceding unsigned comment added by 67.175.112.132 (talk) 16:47, 20 March 2016 (UTC)[reply]

Estimated plate motion
As the adjoining map indicates, the width of the Pacific is shrinking at a rate of around 2 inches per year. If it kept that up for 300 million years it would eventually narrow to nothing. Most geologists probably don't expect it to disappear, though -- the forces that drive plate motion are so poorly understood that it is essentially impossible to extrapolate more than 50 million years into the future. Looie496 (talk) 17:14, 20 March 2016 (UTC)[reply]
What about the rise in sea levels increasing the size by drowning low-lying islands? Dbfirs 20:44, 20 March 2016 (UTC)[reply]
1) The rise is sea levels is due to human activity (global warming), so what the sea level will be in 300 million years is anyone's guess.
2) Higher sea levels would delay the closing of the Pacific, but eventually the mountains on both the Americas and Asia/Africa would crunch into each other, if we assume the plate movements stay the same. StuRat (talk) 20:52, 20 March 2016 (UTC)[reply]
Some concepts for future plate configurations include Amasia, Novopangaea, and Pangea Ultima. The first two of these imagine that most of the Pacific plate is eventually subducted over the next ~200 million years. The last of these envisions an eventual reversal of current trends that spares much of the Pacific and destroys most of the Atlantic. To quote Christopher Scotese, these future scenarios are "all pretty much fantasy to start with". Truth is, we don't really know what is going to happen. Dragons flight (talk) 20:47, 20 March 2016 (UTC)[reply]


Mark Twain noted that the Mississippi River was getting shorter each year - and that it was once a million miles long, and in the span of a few centuries would be only a bit longer than a mile. Extrapolation of such matters does have problems. Collect (talk) 23:23, 20 March 2016 (UTC)[reply]

Had his prediction been about the Colorado, he would have been onto something. ←Baseball Bugs What's up, Doc? carrots23:59, 20 March 2016 (UTC)[reply]
Assuming it is the same reference that I know of, Twain was upset by the dams and dredging of the Mississippi River to keep it from moving around. One of the results was that the end delta was shrinking. So, Twain joked that the shrinking would eventually cause the Mississippi to shrink to nothing. 209.149.114.215 (talk) 16:19, 22 March 2016 (UTC)[reply]
The folks with dwellings on the cliffs which are being undercut by erosion might argue that it's the other way around - that it's the coast that's diappearing. ←Baseball Bugs What's up, Doc? carrots01:24, 24 March 2016 (UTC)[reply]

Preparing Hydrogen cyanide without platinum.

I want to make Hydrogen cyanide for private reason at home or somewhere. Platinum is very expensive, I can't buy it so I need a replacement for that. Platinum is a catalyst. I need something cheap alternative. For more information see https://en.m.wikipedia.org/wiki/Hydrogen_cyanide#Production_and_synthesis thanks — Preceding unsigned comment added by 45.79.180.120 (talk) 23:08, 20 March 2016 (UTC)[reply]

Buy lots of apricot kernels. No further information on such shall be supplied. Collect (talk) 23:20, 20 March 2016 (UTC)[reply]
I know a couple of syntheses, but before I tell you anything I need to know what you plan to use this stuff FOR! 2601:646:8E01:515D:E082:2C68:8C6B:47A9 (talk) 23:21, 20 March 2016 (UTC)[reply]
I think it's a little pointless to try to keep someone from finding out how to make cyanide. It's too easy (if you don't sweat efficiency, safety, and quality control), and the information is too available. I'm not going to support burning books (or gatekeeping access to them) to get rid of dangerous knowledge. I'm also not going to be the one to answer the question, though. --Trovatore (talk) 23:27, 20 March 2016 (UTC)[reply]

OP, you should be aware that, even if your ultimate purpose for the cyanide is not criminal, the unlicensed manufacture may be, and if so would expose you to a part of the US Criminal Code that is treated with a singular humorlessness in the post-9/11 world. Someone can probably find links to the specific statute. --Trovatore (talk) 23:50, 20 March 2016 (UTC)[reply]
The Toxic Substances Control Act of 1976 is probably the most relevant US statute. Tevildo (talk) 00:10, 21 March 2016 (UTC)[reply]
Hmm, that looks like mainly a matter of business regulation, not something people ordinarily wind up in jail for. I was thinking more of "destructive device"-type laws. What all was Shannon Richardson charged with? She actually mailed the ricin, and that was presumably the biggie, but was there any charge just for manufacturing it? --Trovatore (talk) 02:04, 21 March 2016 (UTC)[reply]
Richardson was convicted of "possessing and producing a biological toxin", contrary to 18 U.S.C. § 175. Cyanide is certainly a toxin (although not necessarily a biological toxin), as defined by 18 U.S.C. § 178. Tevildo (talk) 09:26, 21 March 2016 (UTC)[reply]
And there have been direct convictions for KCN. It's a chemical weapon covered by treaty and enshrined in various 18 U.S.C places (that's where most of the statutory crimes are outlined.) 10 years for a chemist possessing grams in his kitchen. Providing material support is undoubtedly a crime as well as a bad idea and I don't think any editor wants to test what the G considers "material support." --DHeyward (talk) 03:59, 23 March 2016 (UTC).[reply]
You have a ref for that? (I suppose you mean NaCN or KCN?) --Trovatore (talk) 04:15, 23 March 2016 (UTC)[reply]
Sorry, KCN (fixed). Ref [7]. --DHeyward And the sidebar pic here is priceless [8]. Like many things like "weapons of mass destruction" the law applying to civilians is much stricter than treaty. The OKC bombing was a statutory WMD attack, but is a routine explosive during war. We drop that size bomb on ISIS without batting an eye. (talk) 05:12, 23 March 2016 (UTC)[reply]
That's messed up. He didn't hurt anyone and he wasn't going to, except maybe himself, and he didn't know he'd done anything illegal, except maybe petty theft, and he goes away for eight years??? That's just wrong. --Trovatore (talk) 05:57, 23 March 2016 (UTC)[reply]
Not really. Fed penalties are generally more severe than state penalties even though it's likely illegal in both state and federal law. Prosecuting a PhD in chemistry and instructor under federal law is more of a "you should know better and in case any other Chemists get this idea, we will put you away for a long time." The Orkin high school dropout making his own bug spray probably doesn't merit a fed prosecution. I suspect a widely available website, like Wikipedia, would be subject to the "let's make an unequivocable example so they don't ever do it again" especially if an editor has both practical knowledge and experience. Do no harm is a fundamental principal and giving out cyanide formulas would seem counter to that (and the law) --DHeyward (talk) 07:38, 23 March 2016 (UTC)[reply]
Yes, really, this is wrong. Morally. He did no harm and the state had no moral right to do harm to him. --Trovatore (talk) 16:07, 23 March 2016 (UTC)[reply]
@Trovatore: That's not quite how harm works. Or rather that's not quite how society has defined culpability for criminal acts. We arrest drunk drivers all the time before they actually hurt anybody because "negligence" is a culpable mental state for a crime when the person is aware of a substantial and unjustifiable risk to other people. Making a conscious decision to commit an act that a reasonable person would consider a substantial or unjustifiable risk is a crime. There are different culpable mental states from negligence to willful malice (like intentional murder) but we don't just let negligence slip by. A chemist that is familiar with cyanide would have a hard time proving that storing it under his sink in quantities that could kill hundreds of people isn't a risk (and he didn't). Drunk drivers have the same problem in court but a "negligent" prosecution is quite different than fatal DUI - both are crimes. Second and third DUI's even without injury face increasing levels of penalty and sometimes culpability. (Oh, and the language used in the US Code - "knowingly" is the culpable mental state. That's a higher standard than negligently and is what would be required element of a successful prosecution). --DHeyward (talk) 05:50, 24 March 2016 (UTC)[reply]
And the Statute he violated was 18 U.S. Code § 229 - it shall be unlawful for any person knowingly— (1) to develop, produce, otherwise acquire, transfer directly or indirectly, receive, stockpile, retain, own, possess, or use, or threaten to use, any chemical weapon; or (2) to assist or induce, in any way, any person to violate paragraph (1), or to attempt or conspire to violate paragraph (1). --DHeyward (talk) 05:35, 23 March 2016 (UTC)[reply]
Which New Jersey landfill are you planning to dump the bodies in? ←Baseball Bugs What's up, Doc? carrots23:58, 20 March 2016 (UTC)[reply]
Looks like it is Newark, NJ, corner of Market and Broad streets, near McDonald's. --AboutFace 22 (talk) 14:09, 21 March 2016 (UTC)[reply]
I have to agree with Trovatore that censorship issues are frightening some of us into silence. I actually raised this issue here recently. The weird thing is that people feel relatively safe to simply add manufacturing instructions to articles - it's a little like the shops in so many cities where they can sell bongs all day long, but if a person comes in and says they want a pipe to smoke marijuana, the store is at risk of losing everything if it sells to him. So if people resent this kind of worry - and the agents who might make it more relevant to our community - then the proper response is to improve a broad range of encyclopedia articles to make sure that everyone knows the stuff that "you're not supposed to know". Wnt (talk) 18:02, 21 March 2016 (UTC)[reply]
What "personal use" could someone have for this chemical? ←Baseball Bugs What's up, Doc? carrots20:06, 22 March 2016 (UTC)[reply]
No, sorry. Not going to happen. This isn't anarchists cookbook and it would be foolish to provide any information beyond what's in the encyclopedia. --DHeyward (talk) 20:46, 22 March 2016 (UTC)[reply]
You're right. I just wondered if there is any legitimate "personal use" for this chemical, as opposed to murdering someone. ←Baseball Bugs What's up, Doc? carrots21:05, 22 March 2016 (UTC)[reply]
Hydrogen cyanide, not so much. Potassium cyanide can be used in the Collodion process (wet-plate photography). And, of course, for destroying wasps' nests. Tevildo (talk) 22:49, 22 March 2016 (UTC)[reply]
Coincidentally, I was just looking over some things I had posted some time ago in another venue, and I found this link, where there is a fascinating bit that recommends washing "dishes" (this seems to be some specialized photographic meaning of the word "dish", and not, I hope, something you would eat off of) with potassium cyanide. There is another letter on the same page about "cyanogen soap". It seems that mid-19th-century photogs had a thing about using cyanide to get stuff clean. (My guess is they were trying to remove stray silver, by converting it to soluble silver cyanide, but I'm only guessing.) --Trovatore (talk) 01:25, 23 March 2016 (UTC)[reply]
Whoops, according to our article, silver cyanide is not soluble. In that case I dunno. Would be happy to hear informed commentary or even good guesses with good reasons behind them. --Trovatore (talk) 01:31, 23 March 2016 (UTC)[reply]
Suicide, of course. 78.148.107.251 (talk) 23:22, 22 March 2016 (UTC)[reply]
Let us know how it works out for you. ←Baseball Bugs What's up, Doc? carrots23:54, 22 March 2016 (UTC)[reply]
Suicide kits that are a) less painful and b) more easily attainable and c) doesn't kill first responders and neighbors, and are readily available. If you want to check out, go research it. --DHeyward (talk) 07:57, 23 March 2016 (UTC)[reply]
@Baseball Bugs: In the erstwhile Land of the Free, an older person I know had a little can of potassium cyanide, and would take part of a spoonful and put it on the entrances to yellowjacket nests. The yellowjackets would drop like flies as they came in or out until none were left. Apparently this was much better performance than the insecticides of the time, which by now are probably also banned for being poisonous. (At least with cyanide, you know if you walked away OK you don't have to worry ten years later) Wnt (talk) 11:09, 23 March 2016 (UTC)[reply]
When someone is vague about how they intend to use a toxic substance, it invites scrutiny. ←Baseball Bugs What's up, Doc? carrots22:32, 23 March 2016 (UTC)[reply]
Why are you all answering a question that clearly requires legal advice? --Wirbelwind(ヴィルヴェルヴィント) 17:35, 23 March 2016 (UTC)[reply]
The person asking the question is not asking for legal advice. The issue coming up is whether Refdesk volunteers need legal advice. Note the talk page thread I linked above, which was closed well before this was asked. At the time I was suspicious that some agent of some government might be looking to try to make a trumped-up prosecution of Refdesk regulars... this paranoia has not decreased lately. And however unlikely, it seems orders of magnitude more likely than a genuine criminal let alone a terrorist coming here to ask. (I say this because Roskomnadzor is after our meth articles, and Dianne Feinstein can't be far behind) Wnt (talk) 00:39, 24 March 2016 (UTC)[reply]
The last process mentioned in the section you link is a simple way to make hydrogen cyanide. But the required input is an alkali metal cyanide. Graeme Bartlett (talk) 22:20, 23 March 2016 (UTC)[reply]

March 21

Is that true that vegetables as well as eggs lose their vitamins while cooking or frying?

I was told that vegetables as well as eggs lose their vitamins while cooking or frying. The one who told me it explained me that the cooking or frying causes to the denaturation which cancel the vitamins. Is that true or just speculation? 93.126.95.68 (talk) 03:10, 21 March 2016 (UTC)[reply]

I believe it depends on the specific food, and the cooking temperature and duration involved, and which nutrient we are talking about. Some are indeed reduced by cooking, while others may be enhanced or made more readily available. And of course, some foods are dangerous to eat uncooked. Raw eggs, for example, may contain salmonella. StuRat (talk) 03:22, 21 March 2016 (UTC)[reply]
Thank you. I'm not talking about something specific, but generally about foods. Regarding to the eggs, sometimes there is no option to cook it before eating (for example if we make frosting which made of uncooked eggs white) 93.126.95.68 (talk) 03:30, 21 March 2016 (UTC)[reply]
Well, the statement as given is pretty much false, in that denaturation relates specifically to proteins, not vitamins. Any degradation of vitamins by heat is due to some process other than denaturation. - Nunh-huh 05:12, 21 March 2016 (UTC)[reply]
I believe denature also has a broader meaning, meaning simply "to change the nature of", as in denatured alcohol. Perhaps they are using that meaning. StuRat (talk) 06:11, 21 March 2016 (UTC)[reply]
Anyway, here's some references:-
  • Effects of different cooking methods on health-promoting compounds of broccoli ("The results show that all cooking treatments, except steaming, caused significant losses of chlorophyll and vitamin C and significant decreases of total soluble proteins and soluble sugars.").
  • The effects of cooking on nutrition ("Heating affects mostly the vitamin and fat content of foods... On the other hand, cooking processes that involve heating also make certain nutrients more available for the body to use. For example, the amount of total carotenoids content in carrots and other vegetable-based dishes is higher in boiled versions.").
  • Effects of different cooking methods on the vitamin C content of selected vegetables ("This study shows that any raw vegetable contains the highest content of vitamin C compared to that of cooked one. Eating raw vegetables is the best way to obtain vitamin C. Cooking methods (i.e. steaming, microwaving, and boiling) have huge impacts on the vitamin C content of vegetables. Steaming is the best cooking method for retaining the vitamin C content in vegetables.").
  • European Food Information Council - The Why, How and Consequences of cooking our food ("Part 2.4: ...The differences in vitamin solubility mean that the method by which foods are cooked has a substantial influence on the final vitamin content. Due to their tendency to disperse in water, water-soluble vitamins in particular are heavily affected by cooking processes that involve immersing food in water for long periods of time e.g., boiling. In contrast, fat-soluble vitamins tend to be lost during cooking processes where foods are cooked in fat e.g., frying, or when fat is lost from the product e.g., grilling.").
So the answer seems to be yes, although this varies according to the cooking technique and the type of vitamin content. Alansplodge (talk) 11:14, 21 March 2016 (UTC)[reply]
However, in the case of boiling, since the vitamins leach into the water, the cure is to drink the water, as in soup. StuRat (talk) 16:58, 21 March 2016 (UTC)[reply]
You wouldn't have said this if you'd read the sources: "As well as the cooking medium, the length of heating can also affect the vitamin content of foods. Both fat-soluble and water-soluble vitamins are susceptible to heat, with the latter being particularly sensitive.""--TMCk (talk) 18:03, 21 March 2016 (UTC)[reply]
I already mentioned that in the first reply. My point is that for foods that really do need to be cooked, for food safety reasons, or to make them palatable, boiling is a much better choice, so long as you then drink the water. Steaming is also good, but frying is bad all the way around. As far as the reduction in nutrition from the heat, eating more can compensate for that. Since veggies have few calories, you can eat quite a few. StuRat (talk) 18:32, 21 March 2016 (UTC)[reply]

And e coli. Cook your vegetables as you would cook your beef because the same animals fertilize both. Cooking and 8 billion people are not coincidence. --DHeyward (talk) 09:03, 23 March 2016 (UTC)[reply]

With eggs you actually lose vitamins by not cooking - see egg white injury. With other vegetables I'd need to see a specific analysis - breaking down the tissue might release some components to be absorbed, and might allow others to become soluble in cooking water that is disposed of, and might even damage some chemically (though I'm not sure what) ... it really needs an empirical test for a specific substance. Wnt (talk) 11:20, 23 March 2016 (UTC)[reply]

What is the substance in eggs which cause them to be foamed while mixing (and at all)?

1) What is the substance in eggs which cause them to be foamed while mixing (and at all)?

2) Are detergents considered foamed substances or they are not? Because I always see that soap has foam. If it is something that is added to the soap, what is the reason for that? Does it help in cleaning? 93.126.95.68 (talk) 03:23, 21 March 2016 (UTC)[reply]

1) Albumin causes foam. StuRat (talk) 03:25, 21 March 2016 (UTC)[reply]
2) Not directly answering your Q, but note that soap and detergent are two different things. StuRat (talk) 03:48, 21 March 2016 (UTC)[reply]
Sodium laureth sulfate is a very common foaming agent and detergent found in bath soaps and the like. EvergreenFir (talk) Please {{re}} 04:12, 21 March 2016 (UTC)[reply]
Thank you for your comment. I always thought that the active ingredient in soap is the detergent. 93.126.95.68 (talk) 04:09, 21 March 2016 (UTC)[reply]
The wiktionary entry (detergent) claims a detergent is a "non-soap cleaning agent, especially a synthetic surfactant". Personally I'm skeptical. I think it's correct to say that soap is properly a detergent, but that in common usage, if you mean soap, you just say soap, so if you say detergent, you usually mean one of the other ones. --Trovatore (talk) 05:26, 21 March 2016 (UTC)[reply]

Ah, the IUPAC Gold Book asserts explicitly that "soaps are ... detergents". I'm not a huge fan of IUPAC and their attempts to uniformize naming, but at least this should provide an example of "detergent" used inclusively. --Trovatore (talk) 05:31, 21 March 2016 (UTC)[reply]
The substances in eggs that cause them to make foam when beaten are proteins that trap bubbles of air - see [9]. Richerman (talk) 07:08, 21 March 2016 (UTC)[reply]
I know this wasn't your question but FYI the water from a can of chickpeas is usable as an egg-white substitute because it foams in the same way. Apparently it's not known why though. 78.148.107.251 (talk) 23:19, 22 March 2016 (UTC)[reply]

Which substance does increase the output while drinking tea or coffee?

I've read in professional article by laboratory professor (here) that tea or coffee cause to increasing in urine output. My question is which substances that are found in these drinks, cause the increasing of the urine output? or by which mechanism if it's known 93.126.95.68 (talk) 04:04, 21 March 2016 (UTC)[reply]

The main substance in coffee or tea that causes increased urination is water. In addition caffeine has a modest diuretic effect, but this appears to diminish with regular consumption. Shock Brigade Harvester Boris (talk) 04:09, 21 March 2016 (UTC)[reply]
Thank you. But when people drink water they don't have increased urination as they have while the drink tea, water or beer (which all of them actually contains less H2O than water) 93.126.95.68 (talk) 04:31, 21 March 2016 (UTC)[reply]
In the case of tea (without sugar or milk), it's got to be over 99% water. StuRat (talk) 05:41, 21 March 2016 (UTC)[reply]
Also, drinking any water-based liquid in sufficient quantity would cause the body to urinate to maintain homeostasis. I think what the article you linked is saying is that tea and coffee will increase urination above and beyond what a comparable amount of water would cause. Shock Brigade Harvester Boris is right that the diuretic effect of tea and coffee is almost certainly due to its caffeine content. EvergreenFir (talk) Please {{re}} 05:57, 21 March 2016 (UTC)[reply]
That's true for coffee, but tea -- black or green tea (Camellia sinensis), not herbal tea -- has a rather strong diuretic effect, largely due to its theobromine content. Looie496 (talk) 14:11, 21 March 2016 (UTC)[reply]
Alcohol (i.e. the ethanol in beer) is, by itself, a diuretic. This is mentioned at Vasopressin#Regulation, see also some popular science coverage [10] [11] and this research article [12]. SemanticMantis (talk) 16:20, 21 March 2016 (UTC)[reply]
Perhaps a bit late to the discussion, but as I recall, metabolites of caffeine (e.g. theobromine) cause vasodilation, increased blood flow to the kidney (afferent arteriole) and renal corpuscle, and an increase in filtration into bowman's capsule. This would elevate urine production. I don't remember if there is an inhibitory affect on Antidiuretic hormone as there is with ethanol. If I had to guess, I'd say no.  Wisdom89 talk 17:44, 22 March 2016 (UTC)[reply]

Airplane susceptibility to winds

Why is that when a plane is airborne, it's more susceptible to strong winds and wind shear compared to standing on the ground with engines shut off (ignoring parking brake)? And why the engines of an airborne plane often can't offset a strong wind, while with engines shut off during parking the airplane is capable to stand still? Thanks. --93.174.25.12 (talk) 08:38, 21 March 2016 (UTC)[reply]

To a large extent the answer is the parking brake you're "ignoring". An aircraft in flight cannot respond to sudden gusts or changes in wind direction because it cannot accellerate quickly enough. At high altitude during cruise it doesn't usually matter very much but during landing or take-off even a small change in direction, altitude, airspeed, and/or attitude can be catastrophic. While parked an aircraft is not as sensitive to wind because it is not moving relative to the ground and the brakes and tyres provide a lot of resistance to movement. In severe weather parked aircraft can also be tied down to the ground for extra security. Roger (Dodger67) (talk) 12:57, 21 March 2016 (UTC)[reply]
(ec) An airborne airplane must maintain controlled Flight by continually balancing the forces of lift, thrust, drag and directional Aerodynamics where every external air movement can cause immediate deviation in any of the flight dynamic angles i.e. pitch, roll and yaw. The control task is so difficult that automatic Fly-by-wire computers are sometimes introduced to replace a pilot's conventional manual flight controls. In contrast, a plane parked on the ground is in a stable equilibrium; it should be safely imobilised against winds by both parking brake and Wheel chocks and its flaps not be left in take-off configuration so that not even a strong wind can lift the plane's weight. There is no need to explain why a plane can park undisturbed in a headwind that its engines would lack enough thrust to fly against. AllBestFaith (talk) 13:03, 21 March 2016 (UTC)[reply]
Wind speed increases with altitude. A plane in flight will (generally) be subjected to stronger winds than one on the ground. Iapetus (talk) 13:48, 21 March 2016 (UTC)[reply]
The original premise is incorrect. Airspeed is airspeed; it doesn't matter whether the relative motion of air with respect to the airfoil it is caused by natural wind or as a consequence of applying engine power to produce thrust. There are a handful of minor details, like propeller wash, p-factor, or slipstream effect - and these only apply to some types of aircraft (check the official pilot's operating handbook for each aircraft!) - but these confounding factors don't change the basic fact: the wing does not know why air is rushing past it. It will produce lift if there is sufficient airflow.
Some of the most difficult and potentially hazardous elements of operating an aircraft are the difficulties taxiing (driving on the ground) during strong winds.
Have a look at the Airplane Flying Handbook, section on taxiing; and next, take a look at the chapter on Transition to Tailwheel Airplane, for some of the maneuvers pilots must learn to stay on the ground, moving in the correct direction.
As far as shear: low level wind shear is hazardous because it can cause an aerodynamic stall at very low altitude as the aircraft escapes the surface zone and the ground effect. The hazard is because at low altitude, there is very little room for error in applying a correction. Abrupt changes in the wind direction or wind vertical component will require careful, coordinated adjustment of the flight control, angle of attack, and/or engine power. Otherwise, wind shear is no different at altitude or on the ground.
For very small and light airplanes, like Citabrias, the airplane stays parked on the ground during a stiff breeze because it is tied down with ropes. If the winds are strong enough... well, here's a famous video from c. 2014. The wind was very strong - exactly strong enough to cause the airplane to fly (with or without the engine running). Some fast-reacting aviators spun up the engines so that they could maintain control of the aircraft and take them out of the area.
Nimur (talk) 16:26, 21 March 2016 (UTC)[reply]
Several redlinks show up in my post: apparently we don't have articles on everything!
Some specific references, for interested readers:
  • Low Level Wind Shear: Invisible Enemy To Pilots (from NOAA), on the science and the history, including a fatal 1985 airline disaster
  • Our article on wind shear and Low level windshear alert system
  • The PHAK, Chapter 11-11, is on Low Level Wind Shear
  • On prop wash and slipstream: the Airplane Flying Handbook, Chapter 4, Slow Flight, and certainly elsewhere in the book, there is extensive discussion of these aerodynamic phenomena...
    • The section on high performance maneuvers and slow flight both discuss the risks of wind and gusts during operations that are close to the critical angle of attack.
    • Read and memorize the unintuitive bits. Unintentionally stalling at low altitude still requires the exact same recovery procedure as intentionally stalling at high altitude - pitch forward. Pilots who encounter abrupt wind shear that causes a stall on takeoff must therefore unintuitively steer "toward the ground" to recover from the stall. (The uninitiated pilot may pull back to fly "away" from the ground, but that's not how airplanes work). Reluctance or hesistation to correctly recover from a low altitude stall will bring about a rapid reacquaintance with the ground. Here is a great historical film, Stalling for Fun and Profit Safety (1974).
  • There probably ought to be separate articles on deep stalls and tail stalls...
Some time later this week, I'll try to create some articles for those redlinks.
Nimur (talk) 16:40, 21 March 2016 (UTC)[reply]

March 22

Dust in air: outside vs inside

Can you reduce dust floating around in the air by opening a window or is there as much dust floating around outside as inside? I don't live near any industrial facilities I would expect to be churning out dust. --78.148.107.251 (talk) 08:52, 22 March 2016 (UTC)[reply]

I think that would depend very much on what is going on both inside and outside. Outside you will often find pollen as well as dust, and you need to take into account the direction and strength of the wind. Inside dust will depend on how much movement there is to churn it up into the air. The answer will be sometimes yes and sometimes no. Air conditioning can have filters to eliminate or reduce the dust that enters the room. Electrostatic devices on the market claim to be able to reduce the dust in the air of a room. Dbfirs 08:59, 22 March 2016 (UTC)[reply]
Depends on weatherconditions and season. On a rainy day the air outside will be near dustfree. In spring and summer you will have weeks when outside air is so filled with Pollen and/or spores you better keep your windows shut unless it is/was raining. --Kharon (talk) 15:37, 22 March 2016 (UTC)[reply]
Desert dust floats around in lots of places - especially in dry climates - and not just the Sahara! For example, here in California, that stuff leaves a mess on the windowsills if you leave the windows open, and it accumulates fast.
NASA has studied atmospheric aerosols and found desert dust at altitudes as high as 15,000 feet above sea level. Not only does natural dust make a mess in the house, it also affects global climate!
Our State Government even publishes this fun comic book, Fugitive Dust Control, from the California Air Resources Board. It's not just a nuisance - it's air pollution! A lot of dust is natural, but human activities like farming, construction, and motor vehicle traffic can significantly aggravate it, especially by getting large particulates up to higher altitudes where natural winds can carry them farther and higher.
Last year, the Pulitzer Prize was awarded to Diana Marcum for her photographic series on the "Dust Bowl" that we've been experiencing for the last few years. We finally got some rain and snow in 2016!
Nimur (talk) 16:17, 22 March 2016 (UTC)[reply]
A lot depends on local conditions. On average, indoor environments tend to be dustier by 20-50% than outdoor environments. So on the typical day, opening the windows will improve indoor air quality. However, day-to-day air quality can often see outdoor dust levels swing by factors of 2 or 3. On a day when the external air is unusually bad, opening the windows may make things inside worse. You also don't have to live near industrial plants to have bad air outside. Contaminant plumes can travel hundreds of miles when conditions are right. Here are a couple maps to help look up your local air quality right now [13][14][15]. On the other hand, certain indoor activities, including smoking, cooking, and using a fireplace, can rapidly increase the level of indoor particulates. (This is a huge problem in third-world areas that use coal, kerosene, or firewood for cooking and personal heating.) If you have resources and the inclination, the best way to reduce indoor dust is with a HEPA filter, either on a central AC / Heating system or as a stand-alone unit with a fan. The electrostatic devices Dbfirs mentioned tend to be more gimmick than actual benefit. Most of those have a poor flow rates and so take much longer to clean the air than filters, and they also generate small amounts of ozone which can be more annoying than the dust depending on how sensitive you are to ozone. Dragons flight (talk) 17:43, 22 March 2016 (UTC)[reply]
Also, it's not just a matter of which is dustier, inside air or outside air, as air passing through a house tends to slow down and drop it's contents. I've noticed that parking in a carport makes my car much dustier than out in the open, for the same reason. So, opening the windows to reduce dust doesn't seem like it would help. On the other hand, opening a window to change the temperature, humidity, and mix of gases inside the house can help a lot, especially with window fans. StuRat (talk) 17:52, 22 March 2016 (UTC)[reply]
In some countries there are flyscreens attached to the windows. It's not just a question of dust, insects can get in as well. 78.149.118.97 (talk) 14:29, 23 March 2016 (UTC)[reply]
I think a large part of your decision should not just be about the quantity of dust - but rather what it's made of and what the particle sizes are. Most indoor dust is dead human skin cells (and pet dander if you have one or more pets) - but outdoor dust can be fine soil particles, carbon from diesel engines, cement particles, pollen, all sorts of things. We know that the very fine dust produced from diesel engines is really harmful - for example - and pollen is responsible for a bunch of allergies. So it might be that by opening the windows, you reduce the total quantity of dust by hugely reducing the amount of harmless dust and actually increasing the quantity that is harmful. That consideration makes this be a tougher decision than just "how much dust" would suggest - and it depends a lot on whether you have allergies and/or respiratory problems such as asthma. SteveBaker (talk) 14:30, 23 March 2016 (UTC)[reply]
Most indoor dust is outdoor dust that came inside: [16][17]. The oft-reported (and rather icky) claim that indoor dust is mostly human skin is simply a myth. Human skin of course contributes, but unless you've got a serious skin condition you aren't going to be the main source of dust in your home. Dragons flight (talk) 14:56, 23 March 2016 (UTC)[reply]

Was just discussing this with a friend. I live in Tucson (desert), 150-feet from a main street, you can see the busses sometimes kicking up dust. In addition to diesel there are fine particles of rubber from the car tires. I asked my friend, who has lived in different parts of Tucson, including next door to me if he thought dust was worse here than other parts, he said, "here." Haven't looked on WP yet for articles on dust but thanks, Nimur for the link to Mineral dust, which is a big problem in Green Valley, 30-miles south of Tucson. Raquel Baranow (talk) 14:47, 23 March 2016 (UTC)[reply]

  • Lets go back to the OP's question and see it if it it makes practical sense to him.... He indicates that he lives outside a industrial area. If he keeps his windows always closed, the air inside may become dryer (unless he is lucky enough to have an old fashion wife that ignore modern frozen veg and instead boils everything on the wood fired range, instead of shoving it all in the microwave). It is better for health to have a reasonable exchange of out side air- especially if he lives in a modern draft free home. The more moist out-side air will neutralize the electro potential of dust, allowing it to settle. Buy a cheap humidity meter – they are actually rubbish and inaccurate but it will make one more aware of the humidity. If ones eyes feel dry or the nose feels not quite right, it becomes obvious that the humidity has dropped too low. So, don't keep your home too dry. Meaning by that the whole family may get colds because their nasal passages have dried up -which happens with British style central heating. If your worried by dust, then take Dbfirs advice. Invest in a air ioniser. Don't by cheap. For one that is any good at all will cost about a hundred Euros but I have one and it was worth it. Finally cat dander. Some people think they have dust problems because visitors wheeze – even though the home owner don't own a cat. Simpler explanation: In your part of the world, don't bother looking for cats. Just get a shot-gun-licence and stay up for a few nights and blast way at any haggises that try to get into you home. Mind you, wear wellys, as their teeth are razor sharp. Don’t take my word for it – talk to your neighbours.. Get one of those critters cornered and their incisor can go the a rubber gumboot like a hot knife though butter. Which is why you never see them (or being allowed ) to kept as domestic pets in England.--Aspro (talk) 21:13, 23 March 2016 (UTC)[reply]
A lot of people have air conditioning, which I guess would purify the air anyway. In Britain, the government is pushing environmental improvements to homes. Mine has been fitted with some kind of extractor (not air conditioning) which circulates the air and draws condensation out through the roof. I've never switched it on so I can't say how it would affect the air quality, but I have the manual (somewhere) which explains how it works. 90.215.70.109 (talk) 21:35, 23 March 2016 (UTC)[reply]
I don't think A/C itself cleans the air. However:
1) Central A/C likely has some air filter(s) to clean the air it circulates. Window A/C units may have filters, too, but in either case they require regular cleaning or replacing to do much good.
2) A/C can generate mold (and mold spores) if the condensate isn't properly drained. Or, even worse, Legionnaire's Disease.
Also, can I ask why you've never switch the extractor on ? Only plan to use it if you burn something badly in the oven ? StuRat (talk) 21:47, 23 March 2016 (UTC)[reply]
@ 90.215.70.109 have you ever ventured into your attic (the space above the top floor ceilings and the roof tiles) to look at the ventilation ducking? If you don’t have an attic just look in the loft. You may find it goes through a sort of box with a power cable attached before venting outside. If so, that will be your economizer that recovers heat -to reduce your winter fuel bills. --Aspro (talk) 23:30, 23 March 2016 (UTC)[reply]
We do have an article on dust. People with specific medical conditions might pay attention to pollen or dust mites. But I am suspicious that there is a lot of hooey put out by people looking to sell noisy equipment. Our ancestors lived in little huts with a fire in the middle and a little hole at the top to let the smoke out, and we've come a long way since then. And for most non-allergy sufferers, an abundance of antigens is just an opportunity to reinforce immunological tolerance. (A wild speculation I favor is that sniffing flowers is a behavioral instinct to induce just that...) Wnt (talk) 23:57, 23 March 2016 (UTC)[reply]

Crude ash content of food

Dog food and cat food labels often indicate the proportion of ash in the food. Our article on kimchi says it contains 0.5 g "crude ash" per 100 g. Our article on ash doesn't say anything about what the term means in reference to food. On the website of an animal feed manufacturer [18] it's explained that crude ash refers to the total mineral content of the food. How do we want to accommodate that information at Wikipedia? Should we have an article on crude ash? Should that term redirect to ash? If the latter, then could someone with a better understanding of the issue than me please add the food-science meaning of ash to the article? —Aɴɢʀ (talk) 21:54, 22 March 2016 (UTC)[reply]

The idea for reporting crude ash is that it is an easy thing to measure: just burn the dog food and weigh what's left over. That "crude ash" measurement is almost entirely made up of minerals from animal sources, though plant ingredients may contribute some small amount as well. Meat and bone meal is the source of crude ash in many pet foods. We also have articles on bone meal and blood meal that may be relevant. Crude ash could also be mentioned at Food_composition_data. SemanticMantis (talk) 22:10, 22 March 2016 (UTC)[reply]
Read this article and come back if you have any questions. --Jayron32 00:50, 23 March 2016 (UTC)[reply]
Fascinating... I always wondered this one too. EvergreenFir (talk) Please {{re}} 06:01, 23 March 2016 (UTC)[reply]
Thanks Jayron I intended to link that article too. I do think OP has a good point, and that this should be mentioned on WP, but I can't figure out where either... I almost did a quick redirect to Blood and Bone Meal, because that is what the term (mostly) means in pet food, but that would be misleading for e.g the kimchi claim, where the crude ash I think must come from traces of plant minerals and the fish. SemanticMantis (talk) 14:15, 23 March 2016 (UTC)[reply]
I think you can redirect the link to mineral nutrient, because that's what ash is. --Jayron32 18:37, 23 March 2016 (UTC)[reply]
Good call, I made the redirect and put an (unreferenced) sentence in near the top; a mild overall improvement IMO. Thanks to OP for bringing this up. SemanticMantis (talk) 20:15, 23 March 2016 (UTC)[reply]

uranium or plutonium in nuclear weapons

I had a read through a lot of very nice articles. And it feels like am now capable to built operate and recycle a pit myself. But the thing which is missing is: How many of the US or Russian nuclear pits now in "use" are made from uranium and how many are plutonium ones? The W88 states in a drawing it has a plutonium pit, while the article on B61 nuclear bomb does not give the information. For the Russian nukes it is even more complicated but the Megatons to Megawatts Program makes it clear that they use a lot of uranium which is later used in US nuclear power plants. (Us do not use MOX so it must be uranium). I doubt that you can plug a U-pit into a B61 and after a few years rip it out and plug a Pu-pit in. If this is not a crucial military secret which is also possible there should be a number somewhere.--Stone (talk) 22:34, 22 March 2016 (UTC)[reply]

I'd be very surprised if this exact info can be tracked down...most US are plutonium though...I'd be surprised if there's anyone in the govt who's even made an exact tally...seriously..68.48.241.158 (talk) 23:09, 22 March 2016 (UTC)[reply]
As the pit article you linked says, the fission pits, or "primaries", in all modern nuclear weapons are plutonium. Uranium is used, not in the "primary", but in the "secondary" of most thermonuclear weapons, for the "tamper" around the fusion fuel. When the weapon is detonated, the uranium tamper absorbs neutrons emitted from the primary and the fusion fuel and fissions. See nuclear weapon design for more details. --71.110.8.102 (talk) 00:43, 23 March 2016 (UTC)[reply]
The first place I went looking was to the website of the Federation of American Scientists - www.fas.org - specifically, to their publications and reports on weapon proliferation and fissile materials. These guys are a reliable source, in the sense that they're a bunch of science experts who are well-respected - but their data are independently researched, so take it all with a grain of salt (especially if you're in a decision-making role with respect to nuclear policy!)
Here is FAS's rundown of worldwide nuclear weapons and fissile material; here is their summary status of nuclear weapon states and their respective capabilities. They have lots of data - but it's all well-educated estimates. Chances are, some of the exact information about total weapon capabilities and compositions is highly secretive - although, maybe to a lesser extent than people might expect. A large part of Cold War détente was structured around elaborate and detailed reciprocal disclosure, and effective auditing, of the technical capabilities and stockpiles of all nuclear-weapons states. For example, see the details in our article on the historical Strategic Arms Limitation Talks, which started things rolling... and of course, the New START treaty that is active today; and you can just keep reading about progress up to the present day, such as the proposed Fissile Material Cut-off Treaty, and so on.
As an example of public disclosure: the well-known "aircraft boneyard" of the 309th Aerospace Maintenance and Regeneration Group at Davis–Monthan Air Force Base is a grand collection of American nuclear-weapons-capable aircraft, all intentionally laid out in nice rows so that Russian spy satellites can easily photograph them and count them - to ensure we've actually decommissioned the correct number of weapons platforms! There's reciprocity - American spies can inspect Russian demilitarization too, and (many of) their platforms and technologies are all thoroughly laid out in the open in accordance with treaties, so that nobody has to go to nuclear war over any cloak-and-dagger style doubts.
Heck, you can even sign up for a training course taught by the Department of Defense to learn all about nuclear weapons capabilities, and how we cooperate with (mostly) Russian scientists to ensure that everyone's capabilities are well-understood and comply with the spirit and the letter of international laws. Here's a great list of links to websites of the United States Government that may provide information of interest.
Finally, here is Nuclear Risk, the website of recent Turing Award winner (and co-inventor of public-key cryptography) Martin Hellman - who is also a board member of the Federation of American Scientists. He's got links to lots of useful stories and data pages.
Nimur (talk) 05:03, 23 March 2016 (UTC)[reply]
If you visit it, you'll also see the planes are cut in very specific ways. There are also 18 old Titan missile silo's where the concret sliding doors are half-open. It allows inspection of the silo without it looking like a launch threat or an operational silo (they use to rank cities in "first strike scenarios, Tucson was high on the list with 18 MIRV silos, and the graveyard. Not all the planes stored are destroyed strategice bomber, most are supposed to be able to be rebuilt. F-4's, F-5's F-111B are all mothballed. B-52's are cut up.). Just north in Pinal county is a CIA facility as wellas a commercial graveyard. Tiny, no tower airport lined with 747's including the NASA 747 used to move the Space Shuttle. Landed their once to see the NASA plane and was escorted away rather quickly to the little restaurant. ) --DHeyward (talk) 16:31, 23 March 2016 (UTC)[reply]
I don't think WHAT we have is particularly secretive...in fact, it's probably the exact opposite, as you suggest...but substantial aspects of how the nuke program is operated are secret, I'm quite sure..all I know is plutonium is what we largely use (exclusively use?) for our nukes....it's less expensive than using uranium (though more messy too)..of course plutonium is made from uranium, but whatever..but whether what we have is plutonium based or uranium based makes absolutely no difference to anybody, including potential adversaries, as the effect is the EXACT same... 68.48.241.158 (talk) 14:27, 23 March 2016 (UTC)[reply]
On Thermonuclear War - a famous book by Herman Kahn - addresses that point in his very first chapter: it is difficult for most ordinary people to distinguish between, say, a war that leaves 20 million dead, and a war that leaves 40 million dead - but if we depart from our emotional response and dispassionately consider these potentialities rationally, it is clear that one outcome is preferable to the other. In other words, if you subscribe to the theory of absolutist catastrophism, and pretend that all nuclear war is exactly the same... you're giving up a lot in the way of strategic planning. Nimur (talk) 15:06, 23 March 2016 (UTC)[reply]
It's more political than strategic. Ask average U.S. person which war was "worse" - Iraq, Vietnam or WWII. These are order of magnitude difference, not even just double. Compare to Rwandan genocide. Or Spanish flu to Ebola. The reality is that the political staging is more consequential. It's why people oppose nuclear energy even though rooftop solar energy will burn more houses down and kill more people. --DHeyward (talk) 16:17, 23 March 2016 (UTC)[reply]
'effect being the same' meaning a 1 megaton bomb is a 1 megaton bomb...doesn't matter if explosive force is created via plutonium or uranium..68.48.241.158 (talk) 15:14, 23 March 2016 (UTC)[reply]
Sure, if you gloss over important details like the different construction cost per unit of damage; or the different long term health effects due to variations in, say, how much Strontium-90 radioactive fallout is produced... or any of a zillion other confounding factors! Again, just because the harm is so huge as to be "unfathomable," or because some details are uncertain, does not mean that a strategic planner can never distinguish between the effects. Nimur (talk) 15:21, 23 March 2016 (UTC)[reply]


but if you're looking for something that breaks it down specifically, like 912 plutonium and 206 uranium or something...I just don't think that's discoverable and would be shocked it it could be dug up somewhere...it is of note, of course, for countries that are attempting to create nukes for the first time as far as intel...as the uranium route and the plutonium route look different etc etc..68.48.241.158 (talk) 15:05, 23 March 2016 (UTC)[reply]
Did you read through any of the links I posted? A group of expert researchers do exactly that in the first source I linked: published estimates of the military stocks of plutonium and highly enriched uranium, broken down by nation, and with pages of additional supporting details. Nimur (talk) 15:09, 23 March 2016 (UTC)[reply]
he wants a breakdown warhead by warhead, that info is not in that article...and probably nowhere to be found..68.48.241.158 (talk) 15:20, 23 March 2016 (UTC)[reply]
A little deeper reading is in order - this is a heavy subject! Status of World Nuclear Forces, which is a free online summary webpage, and cites additional publications with even more detailed breakdown. For example, Russian nuclear forces, 2015 includes your proverbial "warhead-by-warhead breakdown." From the same list of references, here is the famous "Nuclear Notebook" published by the same group of concerned atomic scientists. Nimur (talk) 15:26, 23 March 2016 (UTC)[reply]
Shedding some light on why they do it, here is Counting nuclear warheads in the public interest, (2015), a review article about the history of the Bulletin of the Atomic Scientists public disclosures of warhead estimates. Nimur (talk) 15:53, 23 March 2016 (UTC)[reply]
idk still don't see it on a quick glance...if it's there then answer his question! you may not be understanding his question, which is very specific...he wants to know how many war heads in each current arsenal derive their explosive force via plutonium or via uranium....he's not asking about size of arsenal, or a breakdown of explosive force within each arsenal...68.48.241.158 (talk) 16:08, 23 March 2016 (UTC)[reply]
Some questions aren't answerable "quickly." The data is all there, in painstaking detail, with summary tables and pages of descriptions... anybody who wishes to spend hours poring over it to recategorize it into arbitrarily-defined bins to answer an arbitrarily specific quesiton ("How many are Type A and how many are Type B") may do so. That would be an arduous and boring and ultimately fruitless task - and I'm somebody who enjoys poring over boring data sets recreationally!
Personally, I think it is more productive to simply read what the other experts have written - how they have approached the problem, how they have categorized the warheads ... but if you want to analyze and refactor their efforts - go for it! There is more public information than there is time for a volunteer enthusiast to process all of it.
If it turns out that your categorization of global weapon stockpiles into counts of "uranium pits" and "plutonium pits" is correct, valid, and useful, you can even submit it to peer-review scrutiny and publish it!
Nimur (talk) 16:23, 23 March 2016 (UTC)[reply]
it might be possible that ALL in the current US arsenal are plutonium based...idk...68.48.241.158 (talk) 16:13, 23 March 2016 (UTC)[reply]

okay just talked to somebody: pretty sure entire arsenal of US and Russia are technically "thermonuclear" devices currently..ie not atomic bombs, and it's complicated as some uranium is used in these devices for primers apparently, but the 'pit' itself for these are always plutonium based...68.48.241.158 (talk) 16:30, 23 March 2016 (UTC)[reply]

just noticed btw that 71 answered OP's question right at the beginning of the thread...pehaps why he hasn't been back...missed that post..68.48.241.158 (talk) 18:41, 23 March 2016 (UTC)[reply]

I would not be surprised if the primary not even needed to be a fission bomb, just radiation and neutron generator. The secondary can be lead. Spacing and shape and how much radiation pressure and neutron supplied seem key. An actual primary explosion would require spacing to separate sub critical components as well as spacing to allow the radiation and neutrons to spark the fusion process before primary detonated. Some early types did this though I'm not sure that procedure is still used. It would be much safer to have near-critical amounts of radioactive material and the Lithium/Hydrogen matrix encased in a non-neutron supplying medium. Our article describes the russian 50Mt bomb that used lead instead of uranium. 50Mt was half the yield of a uranium secondary but at that point, what's the need? Bring subcritical plutonium together and the heat can melt the radiation barrier (or remove it through other means). --DHeyward (talk) 19:28, 23 March 2016 (UTC)[reply]

That is awesome! I ask a question go to bed work a little on my little Mars-GC-MS and when I look here only have to pick a pdf to read. Thanks to all, especially the one IP. It sounds he would be able to go down into the basement of his workplace and count the pits himself ;-). What I wanted to do to write into the plutonium article and the uranium article a short statement like the one given here: most are Pu, U is not used in large quantities.--Stone (talk) 20:33, 23 March 2016 (UTC)[reply]

have to be careful though...if talking specifically about the "pit" in modern "thermonuclear" weapons, you're talking about plutonium....looking at the entire weapon itself, however, a significant component is enriched uranium as well...both components are used, and the way it all works together is quite complicated...too complicated for me to understand...if you're talking about more rudimentary "atomic bombs" then you can make a more clear distinction between plutonium kinds and uranium kinds...68.48.241.158 (talk) 20:51, 23 March 2016 (UTC)[reply]
Thanks again. I would only use info with reference, the two articles I what to add this are two Featured Articles so it will take some time before I add the statement that both materials are used even in the same warhead, but the pits are moste likely Pu.--Stone (talk) 21:11, 23 March 2016 (UTC)[reply]

March 23

Hospitals inhibiting cell phone signal

I was at a hospital and noticed that my cell phone was getting no signal despite the hospital being in a major (population in the millions) city of a modern industrialized country with generally excellent cell phone coverage. I'm aware some hospitals have a rule not to use cell phones over concerns, founded or unfounded, over cell phones having a small possibility of interfering with medical equipment, but a nurse whom I was talking with told me something I found surprising: she claimed that the hospital not only had a rule banning cell phone usage, but that I was getting no signal because the hospital was somehow inhibiting cell phone signals to prevent anyone from circumventing their rule. Also, I did notice that I got a strong signal outside the hospital, but no signal inside the hospital, though this is not decisive evidence.

So does this actually happen, urban hospitals somehow inhibiting cell phone signals? If so, how widespread is this practice? And what technique or technology does the hospital use to accomplish it? (Deliberate siting in a dead zone outside the range of cell towers? Walls of some particular shape or material? Cell phone jammers? Some other method?)

SeekingAnswers (reply) 09:03, 23 March 2016 (UTC)[reply]

Laws will differ based on jurisdiction, but in the U.S., according to the Federal Communications Commission's website, "Federal law prohibits the operation, marketing, or sale of any type of jamming equipment, including devices that interfere with cellular and Personal Communication Services (PCS), police radar, Global Positioning Systems (GPS), and wireless networking services (Wi-Fi)." - Nunh-huh 09:28, 23 March 2016 (UTC)[reply]
There' no evidence cell phones cause that sort of problem. Interestingly in Britain they tried to bring in some special phones for the emergency services - and they were shown to cause interference! Also they had bad coverage so were a bad idea. I'd have thought there should be some way to have call interception so only emergency service or other authorized calls are allowed. Surely it should be possible to stop people taking their phones out every three minutes and yapping loudly where they shouldn't -- they can go outside first using those appendages called legs if they want to do that. Dmcq (talk) 10:35, 23 March 2016 (UTC)[reply]
If they wanted to really curb the use of cell-phones, they would block them (not actively jam them). Although I doubt this is a huge issue. It appears to be just a case of making the safe bet. --Scicurious (talk) 22:00, 23 March 2016 (UTC)[reply]
How would you block them? That is the question. —SeekingAnswers (reply) 23:48, 23 March 2016 (UTC)[reply]
I have no idea what the reason is, but the observation is credible and it would be interesting to hear more. For example, you might have been under interfering material (some kind of metal shielding for an X-ray station on the next floor up?). Conceivably, a nearby cell phone tower could refuse to continue a connection with you based on triangulation of your position or by spying on GPS in the phone - I don't know if this software exists but it could certainly be written. In that instance I imagine that More Important People Than You would have some special code or their phone numbers on a whitelist or something. Wnt (talk) 11:27, 23 March 2016 (UTC)[reply]
The hospital could be a giant Faraday cage. Use metal sheeting on (or within) the walls and roof and glass windows with a thin metal coating. The latter may be used anyway, as they provide better thermal insulation. I know of a building that was an accidental Faraday cage. The architect liked a metal sheeting exterior. After complaints of no or bad cellphone reception (depending on provider), they considered installing signal repeaters. I don't know whether they actually installed them. I imagine this type of hospital building may have been popular in the Cold War, as it is also immune to EMP. PiusImpavidus (talk) 11:33, 23 March 2016 (UTC)[reply]
Come to think of it, although most serious medical conditions in a private prison are treated with ibuprofen, prisoners are occasionally brought to hospitals for treatment. This implies that a hospital building's primary purpose is to serve as a prison. Cell phones in prison describes some of the sort of software I was hypothesizing; and I think it is obvious that preventing malingering prisoners from violating incommunicado would be ranked far above any possible medical or personal concern you might have. Wnt (talk) 11:42, 23 March 2016 (UTC)[reply]
Your statement makes no sense. You state that occasionally, a prisoner is taken to a hospital to be treated. Then you state that the primary purpose of the hospital is to be a prison. That literally does not follow logically, since the prisoners are in no way the primary population of the hospital... --Jayron32 13:09, 23 March 2016 (UTC)[reply]
My assumption is that if one prisoner coordinates an escape or calls up and threatens a victim or orders a hit, heads will roll. If tens of thousands of people complain that they couldn't update their relatives on what's happening, heads will not roll. Wnt (talk) 15:05, 23 March 2016 (UTC)[reply]
Wow. In a world full of horses, you really are only capable of seeing zebras. Or unicorns... --Jayron32 18:35, 23 March 2016 (UTC)[reply]
This BBC report on the subject shows that there are definite problems in real hospital equipment from real cellphones. It goes on to point out that the phone has to be really close to the equipment to have an effect. I think the conclusion is that phones should be turned off inside treatment areas - but should be fine out in the corridors and waiting rooms.
OK - so that's the pragmatic approach - but you could understand (given the importance of the matter) that in an abundance of caution, they simply apply a blanket rule. That same BBC article says that 64% of doctors admitted that they use cellphones in the hospital in violation of the rules...and that NO deaths have ever been attributed to cellphone interference with hospital equipment.
This FCC page says that it's illegal to block or jam cellphones - and doesn't mention an exception for hospitals. But let's suppose there were some kind of exception made:
  • Clearly (as User:Smurrayinchester points out), jamming the signal would be a stupid idea because the jammer would (by necessity) put out more power in those frequencies than the cellphone itself).
  • Passively blocking cellphone signals with a Faraday cage around the whole building would actually be counter-productive because the cellphone will gradually increase the power of it's transmissions to the absolute maximum in a failed effort to reach nearby cell towers - so this would be a great way to guarantee the maximum possible radio noise!
  • More intelligent (and probably cheaper/easier/more practical) would be to put the faraday cage around the treatment areas to at least shield radio noise from corridors and such - but since we already know that the phone has to be within a few feet of the sensitive equipment, that would also be pointless since the only problematic devices are those INSIDE the treatment areas.
So I VERY much doubt that this hospital was intentionally either jamming or blocking your cellphone. Being in the middle of a large steel and concrete building - possibly on the margins of reception for some other reason - might be enough to produce the effect you're seeing - but I very much doubt that the hospital authorities had done this deliberately. Large/heavy equipment such as body scanners and X-ray equipment could have blocked the signal - and it's plausible that some kind of electromagnetic emissions from such devices was unintentionally interfering with your phone. Either way, moving a hundred yards from that spot ought to have gotten your better reception. Did you actually try that? SteveBaker (talk) 14:16, 23 March 2016 (UTC)[reply]
Re: "Either way, moving a hundred yards from that spot ought to have gotten your better reception. Did you actually try that?": I didn't stay in a single spot. After hearing what the nurse told me, I got curious and went all over the hospital, and even to different buildings of the multi-building hospital. My cell phone normally has no trouble getting a signal inside large buildings, but at this hospital, I couldn't get a signal anywhere inside the hospital buildings, but could outside. —SeekingAnswers (reply) 17:34, 23 March 2016 (UTC)[reply]
The cell phones in prison article links this product, which apparently is not a cell phone jammer. On the other hand, I found this report of cell phones blocked in a hospital in Ireland due to an adjacent prison with cell phone jamming. Wnt (talk) 15:13, 23 March 2016 (UTC)[reply]
I am not sure that a jammer or Faraday cage would prevent a cellphone from interfering with nearby equipment. Would the phone still transmit (perhaps in short, infrequent bursts) as it looks for a nearby tower? Or would it simply listen for the tower and not transmit a reply until it finds one? --Guy Macon (talk) 15:22, 23 March 2016 (UTC)[reply]
Depends. Sometimes it drops into analog modes which will drain the battery quickly. But I find it strange that this topic has gone on so long only because my closest hospital has built in WiFi for the rooms. Now if your in the MRI room or the Xray room, no cell phones are allowed. The cardiac monitors to the nurses station are wireless now as well. I'm sure they don't want you wearing a cell phone while hooked up to the monitor. They even had cell phone charging stations in the ER waiting room. What they really don't want is employees spending the day on social media instead of patient care. Day care centers have stricter policies than the hospitals. Hospitals (at least in the US) want repeat business so ERs and Maternity wards are the nicest and most accommodating places. --DHeyward (talk) 16:06, 23 March 2016 (UTC)[reply]
This is tangential, but no modern cell phones have an "analog mode". Analog signalling for cell phones was part of 1G systems, which no modern phones support and which have been taken out of service I think everywhere. What will drain the battery is the phone boosting its transmission power if it can't connect to a tower, to try to connect to towers that might be farther away. --71.110.8.102 (talk) 01:30, 24 March 2016 (UTC)[reply]
@71.110.8.102: - Meh, I have an old phone. But seriously, SMS is one of the oldest formats used in pagers ans uses the AT modem command set. Certainly later GSM and 4G LTE have made it digital but I still believe the U.S. with Verizon and others that avoided GSM initially still have some analog service in rural areas and for compatibility a TDMA phone can drop into analog mode. I haven't looked in a while but see if any of those carriers still support TDMA (Qualcomm took over the market with 4G CDMA and FRS is all but gone but I think some of phones still support analog for SMS even if they can't make a call). Pagers still exist. If your phone works in the U.S. but not Europe or Japan, I think it still has analog communication capability. China went CDMA but I think they avoided paying Qualcomm. CDMA licensing dominates Qualcomm earnings. --DHeyward (talk) 07:43, 24 March 2016 (UTC)[reply]
Our Advanced Mobile Phone System and History of mobile phones article suggests all analog or digital AMPS was shut down in the US by 2008. As for pagers, our article suggests there are a whole host of different systems used both local and wide and that these systems are usually distinct from mobile phone systems (which is an advantage for numerous reasons). It's not clear to me if any of them are analog, but neither that article nor our Short Message Service article suggests SMS was ever used for pagers (the SMS originated with GSM not with analog networks). SMS may be used for message submission and partly analog systems may have been used to submit SMS (e.g. Telelocator Alphanumeric Protocol) but those are different issues. Getting back to pagers, our article suggests FLEX protocol is the most common one used in the US nowadays at least for wide networks and it seems to be digital. Nil Einne (talk) 18:41, 24 March 2016 (UTC)[reply]
It's quite possible the hospital may have taken counter-measures that are counter-productive. The person(s) they assigned the task of ensuring that cell phones can't be used in the hospital may not even be aware of the reason. I like to use the example of speed bumps, which are designed to slow traffic down, to make roads safer. However, when you combine people who see them swerving to avoid them and those who don't see them losing control of their car when they hit them at high speed, they may very well make the roads less safe. StuRat (talk) 17:54, 23 March 2016 (UTC)[reply]
No matter where I stand in my apartment, my smartphone registers a low level signal (only two bars), but if I initiate a call, the level immediately increases to maximum (all bars). This implies that the cellphone towers transmit at low level when idle, but increase power when actually interacting with a device. It might be interesting if the OP had initiated a call while inside the hospital - it may have been successful. Most users simply start a call without looking at the level; by checking it he may have been unnecessarily discouraged. Akld guy (talk) 21:24, 23 March 2016 (UTC)[reply]
I would expect that cell phones would then adjust the signal meter depending on if it's during a call or not. Perhaps yours has poor software. StuRat (talk) 23:38, 23 March 2016 (UTC)[reply]
I do not understand your response. It does not seem to bear any relationship to my suggestion that the cellphone tower increases power. Are you saying that my cellphone should misrepresent the signal level that it receives? This is a 6-month-old Samsung 5 with all available updates installed. Please clarify what you meant. Akld guy (talk) 02:18, 24 March 2016 (UTC)[reply]
The cell phone knows if it is currently making a call or not. If it's customary for cell towers to increase power during a call, then the cell phone manufacturers would adjust for that in the signal bar, say by showing twice as many bars, when the phone was not in use. This would make customers happier, since they would see more bars. (Of course, they wouldn't want them to always see full strength bars, as then they would be pissed when they couldn't make a call, and call customer service and complain that their phone was broken.) Think of a voltmeter which has different settings with different sensitivities displayed on the meter. StuRat (talk) 02:28, 24 March 2016 (UTC)[reply]
Utter nonsense, sorry. You're asking me to believe that a cellphone that hadn't initiated a call via a celltower could predict the full-power output of that tower? Not only that, but whatever level it predicted would be a misrepresentation. C'monnn. Akld guy (talk) 02:54, 24 March 2016 (UTC)[reply]
If there is some standard practice, like cell towers boosting to twice the strength during calls, then the cell phone makers would know that and use it. And it's not misrepresentation, since there are no units listed on the bars. The usual interpretation is that it's a percentage of max. If you have 5 bars possible, then 4 bars is 80% of max, etc. So, if they know what the max cell tower output is under each scenario (during a call or not), 4 bars should be 80% of that max. StuRat (talk) 16:28, 24 March 2016 (UTC)[reply]
I've been in our local hospital many times and I can get a signal in only a few areas. I thought that it was probably due to all of the steel used in the construction or all of the electronic equipment, but I don't know. Bubba73 You talkin' to me? 01:53, 24 March 2016 (UTC)[reply]

Is gray matter of CNS - just collection of the bodies?

I'm reading now in a textbook that "Cell bodies are located in the gray matter of the CNS, and their collections are called ganglia in the PNS and nuclei in the CNS." Does it say that the gray matter of CNS is just collection of the bodies and the white matter is the collection of the Dendrites? If it is, what is the reason for the grey color of the bodies? is it because of that the bodies have nuclei?93.126.95.68 (talk) 00:06, 24 March 2016 (UTC)[reply]

The difference in color between gray matter and white matter is largely that the latter contains myelin whereas the former does not. --Jayron32 00:13, 24 March 2016 (UTC)[reply]
A big reason is neuromelanin - see [19]. Indeed, there is a substantia nigra in the brain ... yes, we have all been slandered with a racial epithet. :) Also a locus coeruleus. The source I cite suggests all these wonderful colors are about protecting the brain from free radicals and metals. Quite possibly they're right. But melanin's highly complex structure reminds me of an earlier era when strands of sugars bound to four nitrogenous bases in random order were assumed to have some modest structural role ... I wouldn't rule out the possibility of a big surprise here. Wnt (talk) 00:18, 24 March 2016 (UTC)[reply]
Thank you for your comments. What about the text that I brought? What does it mean to say? 93.126.95.68 (talk) 00:38, 24 March 2016 (UTC)[reply]
It says basically that Grey matter is where the cell bodies of the neurons are located. Concentrations of gray matter are called ganglia in the peripheral nervous system and nuclei in the central nervous system. If you read those articles, you'll learn more about those terms. --Jayron32 01:30, 24 March 2016 (UTC)[reply]
The answers about are pretty much correct. However, I must quibble with the idea that "cell bodies are in gray matter, dendrites in white". This is only the case for neuronal cell bodies, and even then not exclusively so. There are plenty of oligodendrocytic, astrocytic, microglial and endothelial cell bodies in the white matter. Also, whilst neurolaminin is indeed important in giving the substantia nigra its colour, it is not responsible for the making the grey matter grey (it's more a dirty yellow sort of colour in fresh brain anyway, the white matter is indeed fairly white in my experience). Fgf10 (talk) 08:18, 24 March 2016 (UTC)[reply]

Is it known which substance (ingredient) in cow milk causes to sleepiness?

93.126.95.68 (talk) 01:35, 24 March 2016 (UTC)[reply]

This source claims that any protein-rich food should induce drowsiness, so milk can do so, but not exclusively milk. --Jayron32 01:45, 24 March 2016 (UTC)[reply]
Is that also true for turkey? Or is it just because we eat too much of it at one sitting? ←Baseball Bugs What's up, Doc? carrots01:47, 24 March 2016 (UTC)[reply]
Yes, overeating definitely causes sleepiness, as does the excess protein and tryptophan. StuRat (talk) 01:50, 24 March 2016 (UTC)[reply]
Mother Nature's way of directing your body's focus toward digestion? ←Baseball Bugs What's up, Doc? carrots01:55, 24 March 2016 (UTC)[reply]
Basically yes. While tryptophan can cause sleepiness, and tryptophan (as an amino acid) is present in the proteins of milk and turkey (and lots of other foods), the sleep-inducing effects of tryptophan cannot be induced by turkey or milk alone. I believe the source I cited notes that one would need to eat a stomach rupturing 40 pounds of turkey meat to get enough tryptophan to induce drowsiness. Protein-rich foods are themselves enough to induce drowsiness, according to same. --Jayron32 02:02, 24 March 2016 (UTC)[reply]
Yep, like a snake who swallowed a pig. StuRat (talk) 02:02, 24 March 2016 (UTC)[reply]
Isn't it supposed to be warm milk ? That suggests it's a psychological thing, since warm milk is associated with being a baby, much like the white noise of the womb, and the fetal position, things also sometimes used to make you "sleep like a baby". StuRat (talk) 01:54, 24 March 2016 (UTC)[reply]
Google "warm milk and sleep" and many claim it's a myth. However, it also says milk contains tryptophan, as with turkey. ←Baseball Bugs What's up, Doc? carrots01:57, 24 March 2016 (UTC)[reply]
See source above. While enough tryptophan can, of its own accord, induce sleep, you cannot get enough of it through either milk or turkey, or really any foodstuff, to do so. --Jayron32 02:04, 24 March 2016 (UTC)[reply]
Besides the too small amount of tryptophan to cause drowsiness, I wonder if the problem is also that it does not reach the brain when taken orally. Llaanngg (talk) 18:58, 24 March 2016 (UTC)[reply]
Presumably because different things work for different people. As far as finding one universal sleep aid, well, propofol might work, but Micheal Jackson found that's not a wise choice. StuRat (talk) 02:02, 24 March 2016 (UTC)[reply]
Milk is also rather high in sugar content. Hyperglycemia. I used to drink several gallons a week. I gave it up when I was diagnosed as diabetic. Nowadays when I do have a sip it tastes as sweet as fruit juice. μηδείς (talk) 02:32, 24 March 2016 (UTC)[reply]
There is an article analyzing the link between food and drowsiness. Llaanngg (talk) 19:00, 24 March 2016 (UTC)[reply]

The article on "specific impulse" seems to mix up force and mass (physics)

In the article on Specific impulse. It specifies that "If mass (kilogram or slug) is used as the unit of propellant, then specific impulse has units of velocity." this statement may be correct. But! "If weight (newton or pound) is used instead, then specific impulse has units of time (seconds).". Weight is supposed to have unit in mass not force? I suspect there's some serious mixup here. Anyone care to scrutinize this from a physics standpoint ? Ferrofield (talk) 04:01, 24 March 2016 (UTC)[reply]

It's correct. Thrust is force (mass * length/time^2)). If mass is used, the rate of consumption is mass/time. Divide and you get (length/time) which is velocity. Weight, though, is also force (a kilogram is mass, a pound is force - knowing weight is not mass is critical here). (force/(force/time)) = time. It's a measure of efficiency. You are either measuring how quickly the rocket is getting lighter for a given thrust to weight ratio or you are measuring how far the rocket has moved for a given amount of time. --DHeyward (talk) 07:10, 24 March 2016 (UTC)[reply]
Shouldn't it say "If force [N] is used instead, then the specific impulse has units of time (seconds)." ? Ferrofield (talk) 15:25, 24 March 2016 (UTC)[reply]
In physics, weight is a force. Weight can be measured in pounds, which are a unit of force.
In many other contexts, we know that the weight is measured on Earth, so we can easily translate between weight and mass: these two parameters are related by the g (little g) standard gravitational constant.
When people are sloppy, they interchange mass and weight freely. But in our article on specific impulse, we are being precise: weight is a measurement of force. "Pounds" are a unit of force, not a unit of mass.
Some people choose to clarify this distinction by separately defining two different units: Pound (force) and Pound (mass) - but they're really just formalizing a sloppy conversion. To make matters even worse (!) - even if the author intends to distinguish between "pounds force" and "pounds mass", and diligently works to ensure correctness, in common speech many English speakers elide the qualifier and simply say "pound", yielding a linguistic collision.
In our Wikipedia article on specific impulse, and most of our other physics articles, we totally avoid this confusion by using pounds to refer only to force. If you want to use an imperial unit of mass, use the slug (mass). Alternately, you can use SI units.
Nimur (talk) 16:32, 24 March 2016 (UTC)[reply]
You could use "force" but it would be more confusing, I think. People understand intuitively "Thrust to weight ratio." That's a dimensionless term since thrust is Newtons and weight is Newtons (or "pounds"). The "impulse" is thrust to weight ratio with the added "and thrust to weight ratio improves because the rocket is expelling part of its weight in Newtons/sec)." The dimensions work out but what's missing and might be confusing is it's an integral approaching an ideal Dirac delta function. Ideally, a rocket would have infinite thrust and expel all it's fuel in 0 seconds so it doesn't have to lift the fuel. That 0/0 equation has an integral solution. The difference in using mass (slug/kilogram} or weight (pound/Newton) lets the engineer calculate lift efficiency. Using mass, a moon rocket can be compared to an earth rocket as it will be the same efficiency, but using weight shows the difference in the the amount of burn necessary. An identical rocket motor expels mass at the same rate on the moon as it does on earth and has the same thrust but not the same weight. The rocket weighs less on the moon so the thrust to weight ratio is better - that means less burn time and less fuel (i.e. "seconds" of burn). When calculating efficiency and capability, both metrics are useful/needed but you can't lose sight that it's an integral and units aren't independent of the fuel, mass of the rocket or thrust. --DHeyward (talk) 16:56, 24 March 2016 (UTC)[reply]
Tiny nitpick: an identical rocket on the moon has about one atmosphere less back-pressure on the exhaust system - which has a nontrivial effect on the exhaust mass rate and the flow expansion characteristics. To really get into the details, we'd have to do some "rocket science" - but anyone who looks at, say, the iconic bell nozzle of the Apollo Service Module Service Propulsion System - and compares the bell nozzle shape to, say, the F-1 engine, you can almost intuitively feel that one of those engines was designed to exhaust into vacuum, and one was designed to eject exhaust at sea level. For any particular ambient backpressure, there is a particular nozzle expansion shape that will encourage laminar flow for optimal energy and momentum extraction, even at the same Isp. Nimur (talk) 17:44, 24 March 2016 (UTC)[reply]

A reflection on black hole event horizon (physics)

A black hole event horizon is supposed to be the final end of interaction of anything falling into it with the outside world, besides hawking radiation and preservation of information. Now suppose the gravity exerted by the black hole is measured externally and an object less massive than the black hole falls into it. Will the gravity of the black hole plus an object outside of it become less in magnitude when the external objects falls into it? Such that the event horizon has the ability to chop gravitational interaction? Ferrofield (talk) 05:06, 24 March 2016 (UTC)[reply]

No, the mass of whatever falls into a black hole adds to the mass of the black hole. -- BenRG (talk) 09:13, 24 March 2016 (UTC)[reply]
Indeed, the mass (and therefore the gravitational force), the electrical charge and the spin of the incoming object all add to whatever the black hole had before it 'consumed' it. Conservation laws do apply to black holes. SteveBaker (talk) 13:02, 24 March 2016 (UTC)[reply]
So light which is an electromagnetic phenomena with the propagation of 3*10^8 m/s can't break free. But gravity with the same wave nature and propagation of 3*10^8 m/s goes straight through. Kind of like the black hole where selective about which wave types it permits to escape. It's like it's permeable for some type of waves and not others. Perhaps there are other wave types yet undiscovered that would enable one to see what's inside? (if 2800 light years won't degrade the resolution too much) Ferrofield (talk) 15:17, 24 March 2016 (UTC)[reply]
Um, stop right there. Your statement " gravity with the same wave nature and propagation" isn't necessarily an established way to think of gravity. Gravity has not been properly explained by the standard model and thus analogies between gravity and other standard model forces (like electromagnetism) and their gauge bosons like photons or light are not valid. You simply can't treat gravity like light and ask what a black hole does to gravity. While the standard model did not exist at the time of Einstein, he DID have an intuitive sense that gravity as a force did not work like other forces, which is why he treated gravity as a pseudoforce and developed general relativity to explain the phenomenon of gravity without having to resort to treating gravity like light or other phenomena, which causes all sorts of problems. --Jayron32 15:58, 24 March 2016 (UTC)[reply]
That is the problem. When you try to reconcile relativity with quantum effects the equations break down. So now they are developing "string theory". Basically, a string is a one - dimensional entity, one "Planck length" long, which can vibrate in different ways. It is claimed that forces (including gravity), photons and particles simply reflect different ways this string can vibrate. It is further claimed that although there are "anomalies" in string theory (which would make it untenable) all the anomalies cancel out. 90.215.70.109 (talk) 16:46, 24 March 2016 (UTC)[reply]
Also best keep apart gravity itself and that ripple called "gravity wave" (caused by some interaction of bodies of huge mass) which is more precise a ripple in time or time-space, not a ripple in gravity!! --Kharon (talk) 17:03, 24 March 2016 (UTC)[reply]
The equivalence principle implies that you can treat gravity in more or less the same way as electromagnetism for this purpose (at least for a small object falling into a large hole). The Standard Model's gauge forces are actually similar to gravity; mathematically they are like general relativity applied to compact extra dimensions. And electric and gravitational charge are both preserved, so there is no difference that needs explaining here. -- BenRG (talk) 18:26, 24 March 2016 (UTC)[reply]
As SteveBaker said, electric charge (which is the electromagnetic equivalent of mass) is also preserved. It doesn't disappear. There is no difference between gravity and electromagnetism/light in this regard. There are various ways of thinking about this. One way is that the field is an entity unto itself which can't disappear for geometric reasons, much like you can't untie a knot in a rope if you don't have access to the ends. Another way is that you never actually see the object cross the event horizon; it's always just outside, where it can still "emit" the field for you to detect. -- BenRG (talk) 18:26, 24 March 2016 (UTC)[reply]

Entropy. Statistical mechanics. Information ~ Energy

Question Remark
In this source [exergy.se/ftp/exergetics.pdf, pages 70-77] author writes:

Assume a system of N unique particles. The number of allowed states Ω of the system is exponentially depending on N. Let the probability of the j:th state be Pj and the sum of the probabilities of all states to be 1, i.e. the system is in at least one state

The entropy of the system is then defined from statistical mechanics as

...

Let us exemplify by a system of N different particles with 2 possible states each, e.g. 0 or 1. Then we have . If there are no other restrictions then all must be , see the Table below.

Is it correct? We know form Boltzmann's entropy formula that

,

where .

Taking the logarithm of W, we have

.

Applying Stirling’s theorem, we obtain

.

Let

.

Then

and

.

So first, we must add factor (BTW in wiki articles also is absent). Second, is a number of existing states among all particles, not all possible states. So we count only system states, which consist of same set of particle states. E.g. we have 5 particles of which 2 are in state 0 and 3 are in 1. E.g. we have system 00111. Now we can consider only 00111, 01011, 01101, 01110, 10011, 10101, 10110, 11001, 11010, 11100. We cannot compose 00000 or 11111. In this example we have

So why author believes and counts ??? Is it correct?

[ https://en.wikipedia.org/wiki/Boltzmann's_entropy_formula ]

[ http://exergy.se/ftp/exergetics.pdf , pages 70-77]


37.53.37.94 (talk) 20:07, 24 March 2016 (UTC)[reply]

The author doesn't believe this: he defines it. He has explicitly stated that his hypothetical system has N independent variables that may each take only one of two possible states. This is definitional or axiomatic - he constructed the problem so that he could work out the math that follows from it.
This formulation is nearly identical to what shows up in my textbook on thermodynamics, Stowe's Introduction to Thermodynamics and Statistical Mechanics. It's a pretty standard formulation, because it's a simple way to work the math.
I've never heard of this particular author - so I'm not going to vouch for the entirety of everything else he's written (in particular, his unique terminology "exergy" is not commonly used in other books on statistical physics) - but I can recommend several other great books on statistical mechanics if you're interested.
Nimur (talk) 20:10, 24 March 2016 (UTC)[reply]
But with this formulation he calculates entropy wrong way. He must take . If particle states are 0 and 1 , .